Sunteți pe pagina 1din 110

TABLE OF CONTENTS

Introduction for Parents


An Overview of the tests 3
What do my child’s scores mean? 4
What subjects are tested? 4
How can I help my child perform well? 5
Test Taking Strategies for Students in Grade 4 6
Introduction for Students
About This Book 8
Test Taking Tips 8
LANGUAGE ARTS
Reading Vocabulary-Synonyms 11
Reading Vocabulary-Antonyms 14
Vocabulary Development – Context Clues 17
Vocabulary Development – Multiple Meanings 19
Sentence Structure and Word Usage 21
Figurative Language Skills 23
Capitalization and Punctuation 26
Reading Comprehension 28
Spelling 40
Latin Word Roots 42
Answer Key for Language Arts 45

MATH
Whole Number Concepts and Computation 46
Number Sense and Numeration 48
Measurement and Geometry 50
Statistics and Probability 53
Fractions and Decimals 56
Patterns and Relationships 58
Estimation 59
Problem Solving Strategies 60
Logic and Problem Solving 62
Math Computation 66
Answer Key for Math 70

HISTORY-SOCIAL STUDIES
Geography 71
Economics 74
Civics and Government 76
History 79
Answer Key for History-Social Studies 84

4th Grade Edition 1


SCIENCE
Earth Science 85
Life Science 91
Physical Science 97
Answer Key for Science 104

Math Glossary 105

4th Grade Edition 2


Introduction for Parents
On January 8, 2002, President Bush signed into law the No Child Left Behind Act
of 2001. The law asks America’s schools to describe their success in terms of what each
student accomplishes. It requires each state to create its own standards for what a student
should know in each grade level and then test every student’s progress toward those
standards by administering achievement tests in all schools. 1 In connection with this
accountability program, four to five million students are taking standardized tests every
year. The results give teachers, parents and students invaluable information about a
child’s relative academic strengths and the next steps for learning.

An Overview of the Tests

The tests are standardized achievement tests. This means that they measure
attained knowledge or skills, not potential for achievement. The tests are standardized
because all students who take them receive the same test instructions, the same test
questions, and the same amount of time to complete them.
The tests cover many of the fundamental skills are curricular areas that children
learn in school: math, reading, vocabulary, writing, work-study skills, social studies and
science. The test questions, written in a multiple-choice format, reflect a sample of the
material presented in textbooks that are widely used across the country. In answering the
test questions within the general content areas, students will need to be able to evaluate
and apply information, make inferences, and use many critical thinking skills.
Standardized tests perform several functions. They describe a student’s
proficiency within a subject area. They help parents and teachers identify students’ areas
of relative strength and weakness among the subjects they study. They are a means
through which a student’s progress can be monitored from year to year. They give
educators and parents information from which they can plan instruction. Finally, because
they are norm-referenced tests (see explanation below), educators use them to compare
the achievement of their students with others across the nation who have taken the same
tests.
In many schools, scores are a significant factor in determining educational
decisions such as student promotion and retention, mandatory summer school attendance,
class placement, qualification in programs for high achieving students, principal
evaluations, and school accreditation and reorganization. It is not surprising that
standardized tests have become known as “high-stakes” tests.

What do my child’s scores mean?

Norm-referenced tests provide information about how an individual student’s


performance compares with the scores of a reference or “norm group” of students at the
same grade level. The norm group is a large representative sample of all U.S. students
(of different ages; sexes; races and ethnicities; in urban, suburban, rural, public and
private school settings, etc.) who take the test. After a school administers the test, each

1
For more information, visit http://www.nochildleftbehind.gov.

4th Grade Edition 3


student’s scores are compared to those of the national norm group. The result is reported
as a national percentile ranking (NPR). This national percentile ranking shows a
student’s standing within the groups of students in the same grade who were tested at the
same time of year during the national standardization.
A student’s national percentile rank tells the percent of students in the norm group
that got lower scores on a test than the student did. For example, a student achieving a 78
NPR in reading comprehension scored higher than 78% of the students in the norm group
who took that test. This also means that 22 % of the norm group scored higher on the
reading comprehension section of the test. Percentile ranks range from 1 to 99, with the
50th percentile corresponding to the middle ranking or national median.

What subjects are tested?

The tests are different for every grade level. Most students in grade 4 take Level
10. The tests are intended to cover essential academic skills and content that is most
often taught at each grade level. The Level 10 Complete Battery includes the following
tests: 2

Vocabulary - 34 questions to be answered in 15 minutes


Reading Comprehension - 41 questions divided among 2 sessions of 25 and 30 minutes
Language
Spelling - 32 questions to be answered in 12 minutes
Capitalization - 26 questions to be answered in 12 minutes
Punctuation - 26 questions to be answered in 12 minutes
Usage and Expression - 33 questions to be answered in 30 minutes
Mathematics
Concepts and Estimation - 36 questions presented in 2 sessions totaling 30
minutes
Problem Solving and Data Interpretation - 24 questions to be answered in 30
minutes
Computation - 27 questions to be answered in 15 minutes
Social Studies - 34 questions to be answered in 30 minutes
Science - 34 questions to be answered in 30 minutes
Sources of Information
Maps and Diagrams - 25 questions to be answered in 30 minutes
Reference Materials - 30 questions to be answered in 25 minutes

Check with your school administrator to see whether your child will be required to take
the Complete Battery or simply one or two of the tests.

2
If your child takes the Survey Battery of Tests, there are fewer questions presented, and the times allowed
are adjusted accordingly.

4th Grade Edition 4


How can I help my child perform well on the test?

x Support and guide your child through theses test preparation exercises.
When children take the test, it is to their advantage to be able to focus on what
they know, without having the additional challenge of deciphering unfamiliar test
instructions and format. This book is designed to help your child review the content area
skills measured by the test and to practice test-taking strategies that will build confidence
and maximize performance on standardized tests. The exercises are presented in various
formats that will give the students practice in answering questions much like those that
they are likely to encounter on any standardized test. By working through the exercises
in this book, your child will gain invaluable experience in the mechanics and strategies of
test taking in the context of the various subjects presented in the test.
Your child is likely to benefit from discussing with you the thinking processes he
or she used to select or eliminate the various responses to the exercises. Discuss why one
choice is better than another and ask what in the materials points to the correct answer.
The Parent Tips at the beginning of each content section of this book target strategies to
help the student learn critical information or reason through the exercises. It is better to
conduct test preparation sessions on a regular basis throughout the year than to cram right
before test time. Keep the sessions positive by praising your child for effort and result.

x Read aloud to your child and listen to your child read to you.
Students who do well on standardized tests tend to be excellent readers, as all
sections of the tests contain written material that must be quickly comprehended. High
achievers frequently read for pleasure and have a good understanding of what they have
read. You can support your child as a reader by setting aside a regular time for 15-20
minutes of reading each and every day. Encourage the reading of fiction and non-fiction,
and make a point of discussing with your child what he or she is reading, whether it is a
novel, textbook, newspaper, poetry, or magazine. Try to get your child to react to what he
or she is reading, to agree or disagree with it (and explain why), to summarize it or retell
what has happened, to predict what may happen next, to talk about the characters, or link
what they have learned to something they have experienced. Ask your child to make a
note of any new words encountered along the way. Make every effort to use those words
in conversation on a regular basis. Watch those reading and vocabulary test scores soar!

x Reduce any concern your child has about taking the test. Be positive.
Parents can play an important part in preparing their children to take the test
by promoting positive attitudes and expressing support and confidence in their children.
If your child perceives that you think the test will be too hard or not important, he may
not do his best. On the other hand, placing undue emphasis on the test scores can cause
counterproductive worrying.
The wisest course is to strike a balance by encouraging your child to take the test
seriously and to give a good effort. If your child becomes anxious, emphasize that
standardized testing is a normal school activity that helps the school plan programs and
instruction suited to the needs of its students. Explain that no student is expected to
answer all the questions correctly, and that the test scores are just one indicator among
many that will measure achievement. Tell him not to become discouraged if he finds a

4th Grade Edition 5


question too difficult to answer. If he continues on, he will likely come to questions that
he can answer confidently. Finally, remind your child that he is well prepared to do his
best. After all, he has studied the subjects on which he will be tested, become familiar
with the mechanics and strategies of test taking, and practiced answering sample
questions.

Test Taking Strategies for Students in Grade 4

x Learn how to use a “fill in the bubble” answer sheet.


In most school districts, students in grade 4 receive their own test booklet and a
separate answer sheet. Students read the question, decide what their answer will be, and
then mark their answer on the answer sheet by filling in a bubble with #2 pencils. If their
school does not purchase the answer sheets, students will be able to mark their answer
choices on the test booklet by filling in the bubble that corresponds to their answer. In
that case, the bubbling marks are the only ones that they will be allowed to make on the
tests. (Scratch paper is provided for certain parts of the math test. No calculators are
permitted.)
Parents can help their children learn how to quickly and efficiently fill in the
bubble to mark an answer. The mark should completely darken the bubble without going
outside of it. Make sure they do not waste time trying to fill the bubble perfectly.
Children can practice using the answer sheet provided at the end of this book.
Children should understand that there is only one correct answer for each
question, and if they mark more than one answer to any question, the answer will be
scored as incorrect. That means that they must keep their place on the answer sheet,
making sure that the number of the question they are answering matches the bubble they
are marking on the answer sheet. Any mistakes must be totally erased.

x Listen carefully, bubble accurately, and keep up with the pace.


An important aspect of the test is that a student must listen carefully and pay close
attention to the teacher before, during, and after the test. The teacher will give oral
instructions about the test. The teacher’s words are scripted, and the script sets strict
limits on the number of times instructions and test questions may be repeated. Students
must remain attentive, keep track of the number of the question that they are answering,
and make sure to keep up with the pace. It is critical that they follow all directions to the
letter.
The Level 10 tests are timed tests. Students should work as quickly as they can
without making careless mistakes and should avoid spending too much time on any one
question. The more practice questions that your child takes, the better sense he will have
about when he is taking too much time on a single question. The test administrator will
tell the students how much time they will have to take the test, but after that there will be
no announcements about the amount of time that has elapsed or that remains in the
testing period.

4th Grade Edition 6


x Eliminate any unreasonable answer choices and select the best answer.
In a test with a multiple-choice format, it is sometimes difficult to find the “perfect”
answer among the choices given. The first step is to read carefully through the entire
question and all the answers. Then eliminate all obviously wrong choices in order to
concentrate on the remaining ones. Ultimately, it may be difficult to choose between two
choices. In such instances, the student should reread the question carefully. It will likely
contain key words needed to select the best answer. It may be the case that two choices
are factually correct, but one choice more directly answers the question than another.
Consider the following example.

Read the passage and answer the question about it.

Dogs are members of the canine family. They come in a variety of breeds and
hundreds of different shapes and sizes. Each breed has its own special talent. Sheep
dogs herd cattle and sheep. Golden retrievers make good guide dogs for the blind. St.
Bernards are famous for rescuing people in the mountains. Huskies pull sleds to
transport people and supplies over frozen terrain.

1. What is the main idea of this selection?

A. All dogs are canines.


B. All dogs look the same.
C. Dogs make good companions.
D. Different dog breeds have special talents.

To select the best answer, first eliminate the obviously incorrect choice “B.” Although
the remaining choices are all true statements, and even though choice “A” is taken
directly from the selection, only choice “D” addresses the main idea of the passage.
Choice “D” is the best answer.

x Solve math problems on scratch paper. Work neatly and keep organized.
In most instances, students may not make marks in the test booklet. Teachers will
distribute scratch paper for solving problems. It will be important to transfer the problem
accurately onto the scratch paper, align columns, and keep track of what problem is being
solved. This means that the student will need to pay extra attention to neatness,
organization, and accuracy. Time is often a factor in math tests, so it is will be necessary
to work steadily and focus on the task at hand. If the student is really stumped on one
problem, it may be best to give an educated guess, make a notation about the problem
number, and return to it later if time permits. If he decides to skip a problem, be sure that
he skips the corresponding bubble on the answer sheet, as well. Any questions left
unanswered will be counted wrong, so it is best not too spend too much time on any one
problem. There is probably a question waiting down the line that will be much easier to
solve!

4th Grade Edition 7


Introduction for Students

About This Book

In order to know how well children are learning the information they need to
know, students throughout the United States take certain tests every year. The exercises
in this book will help you get ready for the tests. They will give you practice in
answering questions about the things that you will need to know by the end of the fourth
grade.
This book is divided into four sections: language arts, math, history-social
studies, and science. Each section is divided into important practice skill areas within
that particular subject. Look for the box of tips in each section. The tips will explain
special ways to remember the information or will give you help in working through the
exercises.

Test Taking Tips

x Read for 20 minutes every day.


The skills that you use to read are the same skills that will help you to do well on
any test.

x Review or practice a little bit every day.


If you practice or review what you have already learned in class, it will help you to
remember the information when it appears on a test. It is easier and more effective to
learn something in six 10-minute sessions that in one hour-long session.

x When solving math problems, work neatly and keep organized.


Take extra care when you are writing problems to copy the problems correctly and
compute accurately. It helps to work each problem in its own area, separated from
another problem. Be sure to align columns correctly. This will help you to avoid
careless errors.

x Listen closely and follow directions very carefully.


A student who takes the test must listen carefully and pay close attention to the
teacher before, during, and after the tests. The teacher will give you important
instructions about the test. It is important that you follow the directions that you
hear.

x Stay focused.
You will only have a certain amount of time to complete each test. Be sure
to concentrate on the task at hand. Keep working and don’t let your mind
wander. If you don’t know an answer, be sure not to spend so much time on that one
question that you can’t finish the rest of the test. Consider taking your best guess or
skipping the question and coming back to work on it after you have completed the rest of

4th Grade Edition 8


the test. Just make sure that if you skip a problem that you skip that row on the answer
sheet, too.

x In multiple-choice tests, eliminate obviously wrong choices and select the best
answer.
Sometimes it is difficult to find the “perfect” answer among the choices given. The
first thing to do is to read carefully through the entire question and all the answers. Then
toss away the choices that are obviously wrong and concentrate on the remaining choices.
If you find it hard to choose between two choices that both seem right, go back and
reread the question very carefully. Look for key words that will help you focus on the
best choice. Don’t be fooled by choices that are true, but that do not directly answer the
question! Here is an example:

Read the passage and answer the question about it.

Dogs are members of the canine family. They come in a variety of breeds
and hundreds of different shapes and sizes. Each breed has its own special talent.
Sheep dogs herd cattle and sheep. Golden retrievers make good guide dogs for
the blind. St. Bernards are famous for rescuing people in the mountains. Huskies
pull sleds to transport people and supplies over frozen ground.

1. What is the main idea of this selection?

A. All dogs are canines.


B. All dogs look the same.
C. Dogs make good companions.
D. Different dog breeds have special talents.

To select the best answer, first eliminate the obviously incorrect choice “B.” Although
the remaining choices are all true statements, and even though choice “A” is stated in the
paragraph, only choice “D” tells the main idea of the paragraph. Choice “D” is the
correct answer.

4th Grade Edition 9


x Learn to use a “fill in the bubble” answer sheet.
In most schools, students in grade 4 receive their own test booklet and a separate
answer sheet. This means that you will read the question in your booklet, decide what
your answer will be, and then mark your answer on the separate answer sheet. You will
mark your answer by filling in a bubble with your pencil. It is important to darken the
bubble without going outside of the lines, but do not waste time trying to fill the bubble
perfectly. Be sure to practice filling in bubbles on an answer sheet. You can use the
answer sheet that comes at the end of this book.
Be sure that you fill in only one bubble in each row. That means that you must
keep your place on the answer sheet. Take care that if you are working on problem 7 in
the spelling test, you fill in the bubble in row 7 of the spelling section of your answer
sheet. If you make a mistake or need to change an answer, be sure to totally erase your
original mark in that row.

x Take some practice tests so you won’t be nervous.


The more you get used to thinking about what you know and answering questions
about it, the more comfortable and confident you can be. A positive attitude is always the
best way to approach learning and testing. The exercises in this book are designed to
help you learn what you need to know in the fourth grade and to practice answering
questions about it. When you take a test you will be able to say, “I know how to do this.
I do it all the time!”

4th Grade Edition 10


LANGUAGE ARTS

Practice Skill: SYNONYMS

Objective: To evaluate knowledge of word choice and meaning.

Parent Tip: To help your child expand her vocabulary, be sure to read with your child on
a regular basis and discuss new and challenging vocabulary as it comes up. Offer a
simpler, more common word meaning for sophisticated vocabulary. Consider having
your child maintain a vocabulary journal or word box with new words and their
meanings. To develop and expand vocabulary skills, it is important to have your child do
some supported reading at a challenging level. Reading aloud or reading together is best
for this.

Select a synonym to match the underlined word.

Example: A wood pile.

ȅ A. scrap
ȅ B. lumber
ȅ C. plywood
ȅ D. log

The correct answer is “B”. Lumber is the synonym for wood.

1. To buy goods

ȅ A. sell
ȅ B. bring
ȅ C. purchase
ȅ D. send

2. Mix carefully

ȅ J. combine
ȅ K. separate
ȅ L. join
ȅ M. enter

4th Grade Edition 11


3. A victorious team

ȅ A. victory
ȅ B. creative
ȅ C. hard working
ȅ D. triumphant

4. To fly is to

ȅ J. cast
ȅ K. soar
ȅ L. dive
ȅ M. shun

5. Good luck

ȅ A. bye
ȅ B. time
ȅ C. fortune
ȅ D. money

6. A large country

ȅ J. state
ȅ K. city
ȅ L. district
ȅ M. nation

7. Once in a lifetime chance

ȅ A. opportunity
ȅ B. experience
ȅ C. character
ȅ D. performance

8. Choose a book

ȅ J. read
ȅ K. buy
ȅ L. select
ȅ M. edit

4th Grade Edition 12


9. careless mistake

ȅ A. notion
ȅ B. error
ȅ C. blunder
ȅ D. eruption

10. To brag about oneself is to

ȅ J. tattle
ȅ K. persevere
ȅ L. bubble
ȅ M. boast

11. The hero was brave

ȅ A. brash
ȅ B. courageous
ȅ C. weak
ȅ D. strong

12. He was a humble man

ȅ J. modest
ȅ K. modern
ȅ L. pleasant
ȅ M. wealthy

13. To shake is to

ȅ A. rattle
ȅ B. roll
ȅ C. wander
ȅ D. quiver

14. Push gently

ȅ J. pull
ȅ K. shove
ȅ L. yank
ȅ M. nudge

4th Grade Edition 13


Practice Skill: ANTONYMS

Objective: To evaluate knowledge of word choice and meaning

Parent Tip: To help your child expand his vocabulary, be sure to read with your child on
a regular basis and discuss new and challenging vocabulary as it comes up. Offer a
simpler, more common word meaning for sophisticated vocabulary. Compare words
with their opposite meanings. Consider having your child maintain a vocabulary journal
or word box with new words and their meanings. To develop and expand vocabulary
skills, it is important to have your child do some supported reading at a challenging level.
Reading aloud or reading together is best for this.

Select an antonym to match the underlined world.

Example: That puppy is plump.

ȅ A. fat
ȅ B. round
ȅ C. odd
ȅ D. slender

The correct answer is “D”. Slender is an antonym for the word plump.

1. The puzzle pieces can connect.

ȅ A. combine
ȅ B. join
ȅ C. separate
ȅ D. splinter

2. Do not expose the answer.

ȅ J. show
ȅ K. tell
ȅ L. freeze
ȅ M. conceal

4th Grade Edition 14


3. Be sure to send the original.

ȅ A. certificate
ȅ B. copy
ȅ C. document
ȅ D. authentic

4. Mr. Smith will repair the model.

ȅ J. fix
ȅ K. adjust
ȅ L. maintain
ȅ M. wreck

5. Sam can be very sloppy.

ȅ A. messy
ȅ B. dirty
ȅ C. clean
ȅ D. meticulous

6. The cookies were different.

ȅ J. similar
ȅ K. tasty
ȅ L. unusual
ȅ M. unique

7. The policeman said to stop.

ȅ A. wait
ȅ B. hold
ȅ C. proceed
ȅ D. panic

8. The amateur athlete entered the tournament.

ȅ J. professional
ȅ K. inexperienced
ȅ L. experienced
ȅ M. childish

4th Grade Edition 15


9. The expensive gift was lost.

ȅ A. costly
ȅ B. inexpensive
ȅ C. pricey
ȅ D. priceless

10. The rain began to dwindle.

ȅ J. stop
ȅ K. halt
ȅ L. increase
ȅ M. pour

11. Please confirm your answer.

ȅ A. reply
ȅ B. address
ȅ C. state
ȅ D. deny

12. There was the presence of a skunk in the area.

ȅ J. location
ȅ K. absence
ȅ L. viability
ȅ M. critter

13. The day was very cheerful.

ȅ A. happy
ȅ B. bright
ȅ C. gay
ȅ D. glum

14. They used artificial flowers.

ȅ J. genuine
ȅ K. fake
ȅ L. odd
ȅ M. old

4th Grade Edition 16


Practice Skill: CONTEXT

Objective: To evaluate knowledge of word choice and meaning

Parent Tip: Help your children to read the selection thoroughly BEFORE choosing the
best word for the answer. Many students will make errors because they are not aware of
the complete meaning of the selection and stop at the blank to choose a word.

Choose the best answer.

Read the selection and then find the words below to complete the paragraph.

The children had decided it was time to 1. ________ a new tree house. New lumber was
2. ________ as the old wood had become 3. ________ by termites. First, the children
4. ________ the old tree house, trying to 5. ________ reusable nails. Next, they
designed an 6. ________ plan to rebuild. Then, they began the 7. ________ of cutting
and 8. ________ the boards in place. Before they knew it, the tree house was 9. _______
and ready for 10. ________.

1. ȅ A. forfeit
ȅ B. balance
ȅ C. erect
ȅ D. destroy

2. ȅ J. essential
ȅ K. unnecessary
ȅ L. silly
ȅ M. certain

3. ȅ A. stronger
ȅ B. stung
ȅ C. damaged
ȅ D. endangered

4. ȅ J. rebuilt
ȅ K. reconnected
ȅ L. joined
ȅ M. disassembled

4th Grade Edition 17


5. ȅ A. recycle
ȅ B. rekindle
ȅ C. reuse
ȅ D. remake

6. ȅ J. lesson
ȅ K. team
ȅ L. architectural
ȅ M. television

7. ȅ A. method
ȅ B. process
ȅ C. problem
ȅ D. troubleshoot

8. ȅ J. stamping
ȅ K. joining
ȅ L. handling
ȅ M. nailing

9. ȅ A. complete
ȅ B. organized
ȅ C. concluded
ȅ D. obsolete

10. ȅ J. termites
ȅ K. television
ȅ L. occupation
ȅ M. turmoil

4th Grade Edition 18


Practice Skill: MULTIPLE MEANINGS

Objective: To evaluate word choice in multiple meaning situations.

Parent Tip: Help your child to understand that many words in the English language have
more than one meaning. It is important to know how to use the words appropriately. For
example, the word “case” can refer to a case of soda, a case of measles, a legal case, or a
briefcase. Expanding vocabulary in general, will help your child become more aware of
multiple meanings for words.

Choose the best answer that fits both sentences.

Example:

The sun cast a bright ________.


The canoe was very ________.

ȅ A. shine
ȅ B. ray
ȅ C. light
ȅ D. tight

The correct answer is “C”. Light fits both sentences.

1. The car needs a ________ up.


She can sing a sweet ________.

ȅ A. blow
ȅ B. song
ȅ C. melody
ȅ D. tune

2. Paint will ________ up the house.


The tree was a blue ________.

ȅ J. brighten
ȅ K. clean
ȅ L. pine
ȅ M. spruce

4th Grade Edition 19


3. The automobile ________ closes at 4:30.
Did you ________ the roses?

ȅ A. factory
ȅ B. buy
ȅ C. adjust
ȅ D. plant

4. The table ________ decorated the dining room table.


Mr. Smyth sent a ________ with a message.

ȅ J. topper
ȅ K. student
ȅ L. runner
ȅ M. decorator

5. Cake ________ can be messy!


Who is the next ________ in the line-up?

ȅ A. hitter
ȅ B. batter
ȅ C. mix
ȅ D. player

6. Be careful not to ________ the truck into the wall.


The young ________ climbed the rocks well.

ȅ J. pluck
ȅ K. chick
ȅ L. ram
ȅ M. hooves

7. The ___________ did not stick to the wall.


We needed a _____________ to record the movie.

ȅ A. glue
ȅ B. paper
ȅ C. tape
ȅ D. pin

4th Grade Edition 20


Practice Skill: SENTENCE STRUCTURE AND USAGE

Objective: To evaluate knowledge of correct sentence structure


and word usage such as noun-verb agreement.

Parent Tip: To help your children to make correct choices in noun-verb agreement when
writing and speaking, reinforce correct grammar skills and correct grammatical errors
when they occur.

Choose the best answer:

Example: Tim and ________ will be there.

ȅ A. me
ȅ B. I
ȅ C. you
ȅ D. them

The correct answer is “B”. I is the best answer to the sentence.

1. Many children ________ coming to the party tomorrow.

ȅ A. will
ȅ B. have been
ȅ C. are
ȅ D. will not

2. Will ________ need help today?

ȅ J. me
ȅ K. you
ȅ L. us
ȅ M. that

4th Grade Edition 21


3. It is ________ toy.

ȅ A. there
ȅ B. their
ȅ C. them
ȅ D. those

4. ________ you going to join us?

ȅ J. Will
ȅ K. Won’t
ȅ L. Were
ȅ M. We’re

Choose the correct complete sentence.

5. ȅ A. The field trip was a good learning experience.


ȅ B. The field trip were a well learning experience.
ȅ C. The field trip weren’t a good learning experience.
ȅ D. None are correct.

6. ȅ J. Jimmy did good on his test.


ȅ K. Jimmy didn’t do good on his test.
ȅ L. Jimmy did well on his test.
ȅ M. None are correct.

7. ȅ A. Fables is a favorite one of mine.


ȅ B. Fables are favorites of mine.
ȅ C. Fables are favorite of mine.
ȅ D. None are correct.

8. ȅ J. The painters finished his work early.


ȅ K. The painters finished early.
ȅ L. The painters finished there work early.
ȅ M. None are correct.

9. ȅ A. Traveling quickly, the thieves got away.


ȅ B. Traveling more quick, the thieves got a way.
ȅ C. Traveling quick, the thieves got away.
ȅ D. None are correct.

4th Grade Edition 22


Practice Skill: FIGURATIVE LANGUAGE SKILLS

Objective: To evaluate knowledge of figurative language writing


techniques such as simile, metaphor, hyperbole, and
personification.

Parent Tip: Help your children to recognize these writing techniques that enhance
writing. They should be familiar with the difference between simile and metaphor. A
metaphor is a comparison between two things that are not alike. If a comparison uses the
word “like” or the word “as”, then it is called a simile. Hyperbole is the use of
exaggeration, and personification is the giving of human characteristics to non-living
objects.

Example:

Metaphor: Math is a blizzard of symbols.


Simile: Life is like a box of candy.
Hyperbole: We had to wait an eternity to see the doctor.
Personification: The whining teakettle called to us.

Figurative language is defined as the use of a word or phrase to mean something quite
different from its literal or general meaning. The statement, “I can see right through
you,” means something quite different than actually being able to see through a person.

Choose the best category for each statement.

1. An army of ants marched across the picnic table.

ȅ A. metaphor
ȅ B. simile
ȅ C. hyperbole
ȅ D. personification

2. The spaghetti slithered off my fork.

ȅ J. metaphor
ȅ K. simile
ȅ L. hyperbole
ȅ M. personification

4th Grade Edition 23


3. The child was as cool as a cucumber.

ȅ A. metaphor
ȅ B. simile
ȅ C. hyperbole
ȅ D. personification

4. My temper is as hot as a branding iron.

ȅ J. metaphor
ȅ K. simile
ȅ L. hyperbole
ȅ M. personification

5. The race was over in a flash.

ȅ A. metaphor
ȅ B. simile
ȅ C. hyperbole
ȅ D. personification

6. The computer tormented me by flashing unknown statements.

ȅ J. metaphor
ȅ K. simile
ȅ L. hyperbole
ȅ M. personification

7. The cake tasted like rivers of fudge.

ȅ A. metaphor
ȅ B. simile
ȅ C. hyperbole
ȅ D. personification

8. We waited forever for Steven to call.

ȅ J. metaphor
ȅ K. simile
ȅ L. hyperbole
ȅ M. personification

4th Grade Edition 24


9. The sunset was like a giant pumpkin.

ȅ A. metaphor
ȅ B. simile
ȅ C. hyperbole
ȅ D. personification

10. Tom Smith is a gem of a guy.

ȅ J. metaphor
ȅ K. simile
ȅ L. hyperbole
ȅ M. personification

11. The play lasted a lifetime.

ȅ A. metaphor
ȅ B. simile
ȅ C. hyperbole
ȅ D. personification

12. The coughing engine broke down.

ȅ J. metaphor
ȅ K. simile
ȅ L. hyperbole
ȅ M. personification

13. Powdered sugar is light as snow.

ȅ A. metaphor
ȅ B. simile
ȅ C. hyperbole
ȅ D. personification

14. The thunderclouds looked as gray as steel.

ȅ J. metaphor
ȅ K. simile
ȅ L. hyperbole
ȅ M. personification

4th Grade Edition 25


Practice Skill: CAPITALIZATION AND
PUNCTUATION

Objective: To evaluate knowledge of correct mechanics of


writing.

Parent Tip: Help your children learn about correct capitalization and punctuation by
reinforcing rules as you read all types of materials with your child. Take note of
punctuation in newspapers, stories, business letters, and textbooks, and point them out.

Choose the best answer.

Example:

Mrs. Smith and steven went to a museum.

ȅ A. Mrs Smith and steven


ȅ B. Mrs. Smith and Steven
ȅ C. Mrs. Smith. and Steven
ȅ D. No mistakes

The correct answer is “B”. Mrs. Smith and Steven is the best answer.

1. The jackson twins are coming to the party.

ȅ A. The jackson twins are coming to the party?


ȅ B. The jackson twins are coming to the party?
ȅ C. The Jackson twins are coming to the party.
ȅ D. No mistakes

2. How will you get to New york

ȅ J. How will you get to New York.


ȅ K. How will you get to New York?
ȅ L. How will you get to New York!
ȅ M. No mistakes

3. Please select blueberries cherries, and plums at the store

ȅ A. Please select blueberries, cherries and plums, at the store.


ȅ B. Please select blueberries, cherries, and plums, at the store.
ȅ C. Please select blueberries, cherries, and plums at the store.
ȅ D. No mistakes.

4th Grade Edition 26


4. On Thursday, May 5, 1999 school will be closed.

ȅ J. On Thursday May 5, 1999, school will be closed.


ȅ K. On Thursday, May, 5, 1999 school will be closed.
ȅ L. On Thursday, May 5, 1999, school will be closed.
ȅ M. No mistakes

5. The four seasons are spring summer autumn, and winter.

ȅ A. The four seasons are spring summer, autumn and winter.


ȅ B. The four seasons are spring, summer, autumn, and winter.
ȅ C. The four seasons are spring, summer, autumn and winter.
ȅ D. No mistakes

6. You should read a book called By the Great Horn Spoon

ȅ J. You should read a book called “By the Great Horn Spoon”
ȅ K. You should read a book called By the Great Horn Spoon.
ȅ L. You should read a book called By the great horn spoon.
ȅ M. No mistakes

7. Jacob asked “What time is it?

ȅ A. Jacob, asked “What time is it”?


ȅ B. Jacob asked, “What time is it?”
ȅ C. Jacob asked, What time is it?
ȅ D. No mistakes

8. Creative art is the best exclaimed Anne

ȅ J. Creative art is the best exclaimed Anne!


ȅ K. Creative art is the best, exclaimed Anne!
ȅ L. “Creative art is the best!” exclaimed Anne.
ȅ M. No mistakes

9. We saw students paintings in The Kids Book of Art.

ȅ A. We saw students’ paintings in The Kids Book of Art.


ȅ B. We saw students’ paintings in The Kid’s Book of Art.
ȅ C. We saw student’s paintings in The kid’s Book of Art.
ȅ D. No mistakes

4th Grade Edition 27


Practice Skill: READING COMPREHENSION

Objective: To evaluate reading skills using different types of


materials.

Parent Tip: To help your child with reading strategies for test taking, have her read the
associated questions first. This helps to set some background and focus for the sample
she will be reading. Then, she should read the entire selection thoroughly. As she
attempts to answer follow up questions, she should go back into the selection and locate
supporting statements to verify her choice of answer.

Read the selection and answer questions 1-9:

Stacy was ecstatic. She had been waiting for three weeks for this special Saturday
to arrive. Today, Stacy and her family are driving into the country to select their new
Siberian Husky puppy. The entire family is excited about this new arrival.
As they drive to the kennel, Stacy and her sister, Corinne, begin selecting possible
new names. They try out such titles as Fuzzball, King, Champ, Queenie, and Scout. The
girls’ parents encourage them to wait until they choose their new puppy. They don’t
know if they will choose a boy or a girl. They are also reminded that the puppy’s
personality may trigger the ideal name.
At the kennel, everyone is led to a grassy pasture that is fenced off. In the far
corner, there appears to be a mass of fur. As they approach, everyone notices
simultaneously that the mass of fur is really a pile of sleeping puppies. Not three, not six,
but twelve adorable puppies! How would Stacy know which one to pick? As they
roused the puppies, they eagerly sprang to life. Licking, tripping, frolicking puppies were
everywhere. Everyone was giggling as the puppies vied for attention. However, there
were two little ones that stayed off to the side and appeared somewhat shy. Stacy strolled
over to those two and sat down beside them. Before she knew it they had crawled into
her lap and started licking her hand. Stacy quickly realized she had a big problem.
On the drive home, everyone participated in suggesting names for the adorable
pair of furry bookends. Bert and Ernie, Tweety and Sylvester, or Pooh and Tigger were
ideas that came to mind, but in the end, Stacy liked the simplicity of Jack and Jill.

1. Which sentence from the first passage best states the main idea of the selection?

ȅ A. Stacy was ecstatic.


ȅ B. She had been waiting for three weeks for this special Saturday to arrive.
ȅ C. Today, Stacy and her family are driving into the country to select their new
Siberian Husky puppy.
ȅ D. The entire family is excited about this new arrival.

4th Grade Edition 28


2. The word “trigger” most closely means:

ȅ J. a part of a gun
ȅ K. to trip
ȅ L. to make aware
ȅ M. to calm

3. On what day will Stacy get her puppy?

ȅ A. Sunday
ȅ B. Saturday
ȅ C. Friday
ȅ D. Thursday

4. Why do the parents encourage the girls to wait to name the puppy?

ȅ J. Puppies should not be named in advance.


ȅ K. The puppies already had names.
ȅ L. Using the puppy’s personality might help to select a name.
ȅ M. They will pick a name from a list.

5. How many puppies were there to choose from?

ȅ A. 2
ȅ B. 3
ȅ C. 6
ȅ D. 12

6. The term “roused” means:

ȅ J. agitated
ȅ K. awakened
ȅ L. irritated
ȅ M. selected

7. The phrase “vied for attention” means:

ȅ A. begged
ȅ B. tries to be noticed
ȅ C. calls for help
ȅ D. cries out

4th Grade Edition 29


8. What was Stacy’s “big problem?”

ȅ J. She didn’t know which of the twelve puppies to select.


ȅ K. She didn’t know which of the two puppies to select.
ȅ L. She didn’t want to select any of the puppies.
ȅ M. She didn’t know how to pick a puppy.

9. Which conclusion can you draw from the end of the story?

ȅ A. Everyone in Stacy’s family likes puppies.


ȅ B. Stacy was able to select two puppies instead of one.
ȅ C. The puppies were going to a good home.
ȅ D. The puppies will be happier at Stacy’s than at the kennel.

Read the selection and answer questions 10-19:

For the first ten years of his life, Cesar Chavez lived on a small farm near Yuma,
Arizona. He and his family were Mexican Americans who spoke Spanish. They
lived with other Mexican Americans who were very poor and grew the food they ate
on land they had owned. These families were unable to pay taxes, and the
government took their land away. Many of these families became migrant workers
and moved to California. Migrant workers travel from farm to farm picking crops as
they are ready to be harvested.
Cesar Chavez and his family moved to California when he was ten years old. Life
was hard. The migrant families lived in sub-standard housing with no running water.
Their employers were English-speaking farm owners who weren’t very interested in
the problems of the migrant families. Schooling was difficult for children of migrant
workers. Many children worked in the fields and never went to school. Others went
to school sporadically. Cesar had gone to 30 different schools by the time he entered
the eighth grade. Cesar was a determined student and did learn to speak and read
English.
As an adult, Cesar became a leader of migrant workers and fought for their rights.
He encouraged them, and helped them to learn to read and to vote. He formed the
National Farm Worker’s Association, a union to protect the rights of farm workers.
He is well known for leading strikes, such as the grape workers strike of 1965. This
strike was organized to be sure that the farm owners recognized the migrants’ right to
unite.

10. This selection is about:

ȅ J. migrant workers
ȅ K. Cesar Chavez
ȅ L. migrant schools
ȅ M. union rights

4th Grade Edition 30


11. For what reason did the Mexican Americans lose their land to the government?

ȅ A. They were unable to grow enough food.


ȅ B. The government said that Mexican Americans could not own land.
ȅ C. The Mexican Americans were unable to pay taxes.
ȅ D. The Mexican Americans wanted to give their land away.

12. The term “migrant” means:

ȅ J. moving from place to place


ȅ K. living in poor conditions
ȅ L. fruit and vegetable picker
ȅ M. farmer

13. The sentence, “Life was hard,” refers to:

ȅ A. living in hard, rocky areas


ȅ B. being hardheaded
ȅ C. living in a difficult situation
ȅ D. hard work

14. Why do you think the English-speaking farm owners were not interested in the
problems of the migrant families?

ȅ J. The farm owners were lazy.


ȅ K. The farm owners didn’t want to take the time and spend the money to improve
things.
ȅ L. The farm owners were poor and couldn’t help the migrant families.
ȅ M. The farm owners didn’t like the migrant families.

15. Schooling was difficult for migrant children because

ȅ A. their parents didn’t speak English and couldn’t help them.


ȅ B. migrant children moved around frequently and often missed school.
ȅ C. the teachers didn’t want to teach migrant children.
ȅ D. they were frequently sick and missed school.

16. The term “sporadically” means:

ȅ J. frequently
ȅ K. infrequently
ȅ L. often
ȅ M. never

4th Grade Edition 31


17. A union is most likely

ȅ A. two or more workers working together.


ȅ B. a system of farm owners.
ȅ C. an organization that fights for fair work practices.
ȅ D. a factory.

18. What do you learn about Cesar Chavez’s personality in reading this selection?

ȅ J. He was a poor child.


ȅ K. He was a migrant worker.
ȅ L. He learned to read English.
ȅ M. He cared about how he and others were treated.

19. The main idea of this section is:

ȅ A. Migrant workers are often poor.


ȅ B. Cesar Chavez became a leader for migrant workers.
ȅ C. Cesar Chavez often fought people to get attention.
ȅ D. Migrant workers are important to farming.

Read the selection and answer questions 20-30:

There are about seventy-five different kinds of whales. The blue whale is not
only the biggest whale, but also the biggest animal in the world. The smallest of all
whales is called a pygmy sperm whale. The sperm whale is identified by its over-
sized head. A narwhal is a type of whale that has a tooth that grows from its head. It
can grow to be as long as ten feet!
Whales are the largest creatures that roam our oceans. They are categorized as
mammals and give birth to a baby whale called a calf. As with other mammals, the
calf drinks its mother’s milk until it can feed itself.
Whales breathe through a special hole called a blowhole. Water shoots out of the
blowhole as the whale surfaces. When a whale is underwater, it is actually holding its
breath. This is one example of how whales are not fish. A second example is that
when whales are sleepy, they will lie on top of the water and take a nap!
Whales make unusual sounds that they use to communicate with each other. They
seem to almost be singing. Humpback whales are well known for the unique sounds
they make. Scientists are very interested in studying the sounds of the humpback and
learning more about their methods of communication.
Whales are often called gentle giants, but a certain whale is known as the killer
whale. Orcas hunt seals, large fish, and even other whales. They are the black and
white whales that you will find at Sea World. Shamu is an Orca whale that has been
trained to do tricks. As you may have guessed, Orcas are very smart.
Many whales have become endangered species. They had been hunted for their
blubber that was used as an energy source. Today, there are laws to protect whales

4th Grade Edition 32


that have helped them increase in number. Whales are interesting creatures that give
character to our oceans.

20. The blue whale is

ȅ J. blue.
ȅ K. the largest whale.
ȅ L. a type of narwhal.
ȅ M. dangerous.

21. You can identify a sperm whale by

ȅ A. looking for its horn like tooth.


ȅ B. looking for stripes and spots on its skin.
ȅ C. looking for its oversized head.
ȅ D. looking for a hammer shaped head.

22. Which of the following characteristics do only mammals have in common?

ȅ J. They all have skin.


ȅ K. They nurse their young with mother’s milk.
ȅ L. They breathe underwater.
ȅ M. They eat meat.

23. A baby whale is called a

ȅ A. cub.
ȅ B. pup.
ȅ C. pony.
ȅ D. calf.

24. What purpose does a “blowhole” serve?

ȅ J. It allows the whale to breathe.


ȅ K. It helps the whale to float.
ȅ L. It gives out unusual sounds.
ȅ M. It is a play tool.

25. How do whales rest when they are tired?

ȅ A. They sleep at the bottom of the ocean or along a reef.


ȅ B. They sleep in a shallow cove.
ȅ C. They do not rest at all.
ȅ D. They float atop the water’s surface.

4th Grade Edition 33


26. The term “communicate” means

ȅ J. to share.
ȅ K. to talk.
ȅ L. to eat.
ȅ M. to find.

27. The term “methods” means

ȅ A. a process
ȅ B. a system
ȅ C. a procedure for
ȅ D. all of the above

28. Gentle Giant may not be the best nickname for whales because

ȅ J. they are always gentle and kind.


ȅ K. whales are very shy.
ȅ L. some whales are known to be killer whales.
ȅ M. whales are very smart.

29. Blubber most likely is

ȅ A. whale teeth.
ȅ B. whale tail.
ȅ C. whale fat.
ȅ D. barnacles.

30. Which sentence best summarizes the selection?

ȅ J. Whales are gentle giants.


ȅ K. Whales are mammals.
ȅ L. Whales are interesting creatures that roam our oceans.
ȅ M. There are seventy-five different kinds of whales.

4th Grade Edition 34


Read the selection and answer questions 31-42:

While reading Over the Top of the World, by Will Steger and Jon Bowermaster, I
was struck by a section about pollution in the Arctic. Will Steger is well known for his
dogsled adventure across the Arctic Ocean. They traveled by dogsled and canoe sled
during a four-month trek that crossed the ice-covered terrain formed by the frozen Arctic
Ocean. The book explores all aspects of their expedition, as well as intriguing facts about
Arctic life.
The section on pollution caught my eye, as I had a hard time associating pollution
with a frozen ocean mass. But unfortunately, not even the Arctic Ocean is able to avoid
the destructive effects of pollutants. It begins by discussing the patterns of ocean
currents, and how they travel from the mid-latitudes to the Arctic region and back down
again. In these currents, scientists have discovered evidence of pesticides that come from
cities and farms. “In a process known as ‘transboundary pollution,’ these contaminants
enter the atmosphere or a river system and are carried to the Arctic.” Amazingly, or
perhaps not so amazingly, these pesticides are not easily dissipated. The cold climate
acts as a preserving agent. The book cites an example of a chemical that has a warm
climate life of 8 months, and an Arctic life of possibly 40 years.
The greatest concern is that these pesticides enter the food web where they
threaten the local species. These “contaminants” are found in fish. Seals eat the fish.
Polar bears and the local Inuit people then eat seals. The long-term effects of these
pollutants are still to be seen, but they can only be negative. It’s frightening to realize
that the source of these poisons is “thousands of miles away in areas such as India,
Europe, and the United States.”

31. The term “struck” most closely means:

ȅ A. hit on the head


ȅ B. caught the attention of
ȅ C. hit a wall
ȅ D. hand on a clock at midnight

32. Which sentence from the selection best states the main idea?

ȅ J. The section on pollution caught my eye, as I had a hard time associating pollution
with a frozen ocean mass.
ȅ K. But unfortunately, not even the Arctic Ocean is able to avoid the destructive
effects of pollutants.
ȅ L. It begins by discussing the patterns of ocean currents, and how they travel from
the mid-latitudes to the Arctic region and back down again.
ȅ M. In these currents, scientists have discovered evidence of pesticides that come from
cities and farms.

4th Grade Edition 35


33. The term “terrain” refers to

ȅ A. land surface
ȅ B. water
ȅ C. currents
ȅ D. pollution

34. The term “mid-latitudes” is referring to

ȅ J. areas near the poles.


ȅ K. areas near the prime meridian.
ȅ L. the Arctic region.
ȅ M. areas north and south of the equator.

35. Pesticides are also

ȅ A. toxins
ȅ B. poisons
ȅ C. pollutants
ȅ D. all of the above

36. “Transboundary Pollution” can best be explained as

ȅ J. the movement of pollutants from one area to another.


ȅ K. pollution problems related to recycling plastics.
ȅ L. pollution in boundary zones.
ȅ M. illegal dumping.

37. The term “dissipated” means

ȅ A. to build up
ȅ B. to break down
ȅ C. to regroup
ȅ D. to reorganize

38. What effect does cold have on pesticides?

ȅ J. It reduces their life span.


ȅ K. It reduces their cost.
ȅ L. It increases their life span.
ȅ M. It increases their cost.

4th Grade Edition 36


39. Which statement best explains how the pesticides got into the river system in the
beginning?

ȅ A. Companies dumped pesticides directly into rivers.


ȅ B. Rain created a run-off effect that carried pesticides from farmland and cities to
local waterways.
ȅ C. Pesticides are eaten by river wildlife.
ȅ D. Pesticides evaporate into the air and become toxic clouds.

40. What role does the food web play in this problem?

ȅ J. It demonstrates the long-term effects of pollution.


ȅ K. It demonstrates what animals live in the arctic region.
ȅ L. It helps scientists to know which animals live where and eat what.
ȅ M. It shows a chain of producers and consumers and their role in an ecosystem.

41. What is likely to happen if transboundary pollution continues?

ȅ A. The ozone layer will get thicker.


ȅ B. Scientists will go and study Antarctica.
ȅ C. Pollution will increase and animals will be endangered.
ȅ D. The polar regions will have an increase in animal species.

42. Which solution listed below is most logical for this selection?

ȅ J. Change ocean currents so that pollution is not carried to the Arctic region.
ȅ K. The Inuit people should not eat the fish and seal.
ȅ L. Scientists should continue to study the Arctic region.
ȅ M. Improved pollution controls should be put into effect in Europe, Asia, and the
United States.

4th Grade Edition 37


Read the selection and answer questions 43-51:

Bryan had volunteered to bake some cookies for the school fund-raiser. The students
were trying to raise funds for a class trip to the Metropolitan Museum of Art in New
York City. Each student was required to participate in at least one fund-raiser. There
was a bake sale, a car wash, pizza sales, and a “students for hire” program. Bryan chose
the bake sale and the “students for hire” program.
When Bryan got home after school, he started to make the cookies. He had decided
he would make oatmeal-chocolate chip cookies. He had never made them before because
his mom was usually the baker in the family. Bryan figured he could read the directions
and it would all fall into place. What Bryan didn’t plan for was that his mother’s recipe
card had several stains on it that had blotted out some of the ingredients and the
directions. Luckily, he could figure out the ingredients, but the directions were sketchy.
Bryan wrote down what he could read on a separate piece of paper. He wrote:
Work in flour and baking soda to wet mixture.
Last, stir in oats, chocolate chips, and nuts. Mix well.
Next, add eggs, milk, and vanilla. Beat thoroughly.
First, heat oven to 375 degrees F.
Begin by beating butter and sugar until creamy.
Drop dough onto cookie sheet and bake for 9-10 minutes.
Now he had to figure out the correct order. Bryan used his common sense and ended
up with 4 dozen tasty cookies. Bryan began to think that he should provide baking
services as his “student for hire” job skill.

43. Why did Bryan want to bake the cookies?

ȅ A. To make some money.


ȅ B. To help with the school fund-raiser.
ȅ C. To win a trip to New York City.
ȅ D. To share with his friends.

44. About how old is Bryan in this selection?

ȅ J. 5 years old
ȅ K. 9 years old
ȅ L. 15 years old
ȅ M. 22 years old

45. Why was Bryan unfamiliar with the recipe?

ȅ A. He had never had oatmeal chocolate-chip cookies before.


ȅ B. He didn’t like oatmeal chocolate-chip cookies.
ȅ C. He did not know what a recipe was.
ȅ D. He had not baked oatmeal chocolate-chip cookies before.

4th Grade Edition 38


46. The phrase “fall into place” most closely means

ȅ J. something fell
ȅ K. will work correctly
ȅ L. placed next to
ȅ M. break apart

47. The term “blotted” means

ȅ A. fat
ȅ B. spotted
ȅ C. buttoned
ȅ D. greasy

48. The first step in the directions should be

ȅ J. Work in flour and baking soda to wet mixture.


ȅ K. Last, stir in oats, chocolate chips, and nuts. Mix well.
ȅ L. Next, add eggs, milk, and vanilla. Beat thoroughly.
ȅ M. First, heat oven to 375 degrees F.

49. The second step in the directions should be

ȅ A. Work in flour and baking soda to wet mixture.


ȅ B. Begin by beating butter and sugar until creamy.
ȅ C. Next, add eggs, milk, and vanilla. Beat thoroughly.
ȅ D. First, heat oven to 375 degrees F.

50. The third step in the direction should be

ȅ J. Begin by beating butter and sugar until creamy.


ȅ K. Drop dough onto cookie sheet and bake for 9-10 minutes.
ȅ L. Next, add eggs, milk, and vanilla. Beat thoroughly.
ȅ M. Last, stir in oats, chocolate chips, and nuts. Mix well.

51. The phrase “common sense” could be substituted with

ȅ A. sense of taste
ȅ B. basic thinking skills
ȅ C. cooking skills
ȅ D. imagination

4th Grade Edition 39


Practice Skill: SPELLING

Objective: To evaluate spelling skills.

Parent Tip: Help your children expand their spelling vocabulary by exposing them to
rich print. Reinforce and practice difficult spellings such as ei and ie words. The format
that is often used on standardized tests is what may be most confusing to children with
weaker skills. They need to be able to identify both a correctly spelled word as well as a
misspelled word. Students with weaker phonics will fall into the traps designed into a
test.

Identify the word that is spelled incorrectly:

1. A. addition B. addreses C. administer D. admonish

2. J. agricultre K. access L. algebra M. admiral

3. A. arrangement B. architecture C. amusment D. attention

4. J. beautiful K. braged L. boundary M. brief

5. A. biologist B. brouht C. businesses D. bright

6. J. camra K. caution L. chute M. choose

7. A. choice B. clothen C. cruise D. crews

8. J. describe K. diagram L. dezign M. dictionary

9. A. dought B. dollar C. division D. drawn

10. J. eagre K. eagle L. echo M. electric

11. A. equator B. envy C. enviroment D. explain

12. J. field K. feirce L. fault M. favorite

13. A. fourt B. flies C. fragile D. friendship

14. J. genuine K. genuis L. guild M. geology

15. A. geometry B. guesess C. goal D. grudge

4th Grade Edition 40


16. J. happen K. halfs L. haven’t M. heir

17. A. hollow B. hospital C. hurried D. hieght

18. J. insteed K. impolite L. ignorant M. impossbile

19. A. infection B. inguire C. imperfect D. illegal

20. J. justice K. juree L. January M. judge

21. A. juror B. jerk C. jewl D. jammed

22. J. knapsac K. knew L. knight M. knot

23. A. kneer B. kennel C. keel D. kindness

24. J. laid K. large L. layer M. leafs

25. A. lecture B. legth C. lessons D. liar

26. J. license K. limb L. lamb M. loos

27. A. mission B. mispell C. misuse D. musician

28. J. miosture K. months L. mountain M. multiply

29. A. neither B. nieghbor C. neglect D. nurse

30. J. never K. nefew L. niece M. none

31. A. obay B. occasion C. ocean D. only

32. J. often K. official L. orcestra M. oxygen

33. A. paragraph B. pickel C. purse D. peaceful

34. J. pasion K. panic L. photograph M. portable

35. A. quarter B. quickest C. quick D. quist

36. J. quell K. qualm L. quet M. quite

37. A. radio B. raise C. request D. rital

38. J. refferee K. region L. regular M. reign

4th Grade Edition 41


Practice Skill: LATIN WORD ROOTS

Objective: To evaluate knowledge of word origins, word parts,


and their associated meanings.

Parent Tip: Help your children to become familiar with common prefixes and suffixes
that will help them analyze a word and determine its meaning from its parts. As students
associate meaning to parts of words they know, they are better able to evaluate new
words in reading and test taking situations.

Examples:

Common Prefixes and their meanings: Common suffixes and their meanings:
re – means to “do again” or “from -er – one who does
un – means “not” -ful – full of
equa – means “like” or “same” -able – able to
dis – means “not” -ship – the condition of
de – means “separate” -ly – in the manner of
mis – means “not” or “wrongly” -tion – the state of
pre – means “before” -less – without

Latin word roots:


spec – means “look” or “sec”
segregare – means to “separate”
aequalis – means “get ready”
port – means “to carry”
dict – mans “say”
mot – means “move”

Select the best word to match the given meaning:

1. Able to be adored

ȅ A. adoring
ȅ B. huggable
ȅ C. adorable
ȅ D. adored

4th Grade Edition 42


2. In a willing manner

ȅ J. willful
ȅ K. wishful
ȅ L. willess
ȅ M. willingly

3. Not afraid

ȅ A. unafraid
ȅ B. afraidful
ȅ C. afraidship
ȅ D. afraid

4. Not attractive

ȅ J. attractiveless
ȅ K. attracting
ȅ L. ugly
ȅ M. unattractive

5. The state of being a friend

ȅ A. friend
ȅ B. friendly
ȅ C. friendship
ȅ D. friendliest

6. One who labors

ȅ J. laborer
ȅ K. labrador
ȅ L. laborful
ȅ M. laborless

7. Without fear

ȅ A. fearful
ȅ B. fearing
ȅ C. fearness
ȅ D. fearless

4th Grade Edition 43


8. Able to be carried

ȅ J. carriable
ȅ K. portage
ȅ L. portable
ȅ M. carryful

9. One who teaches

ȅ A. teaching
ȅ B. teach
ȅ C. reach
ȅ D. teacher

10. To say what will happen before

ȅ J. dictionary
ȅ K. preform
ȅ L. predict
ȅ M. predicate

11. One who sees

ȅ A. spectacle
ȅ B. spectacular
ȅ C. specializing
ȅ D. spectator

12. Moved far from

ȅ J. movable
ȅ K. movement
ȅ L. motivation
ȅ M. remote

13. One who says what to do

ȅ A. motivator
ȅ B. dictator
ȅ C. dictionary
ȅ D. motivation

4th Grade Edition 44


LANGUAGE ARTS
Answer Key
Synonyms 3. C Figurative 3. B 38. L 20. K
1. C 4. M Language 4. L 39. B 21. C
2. J 5. A Skills 5. D 40. J 22. J
3. D 6. L 1. D 6. K 41. C 23. A
4. K 7. B 2. M 7. B 42. M 24. M
5. C 8. M 3. B 8. K 43. B 25. B
6. M 9. A 4. K 9. B 44. L 26. M
7. A 10. L 5. C 10. K 45. D 27. B
8. L 6. M 11. C 46. K 28. J
9. B Multiple 7. B 12. J 47. B 29. B
10. M Meanings 8. L 13. C 48. M 30. K
11. B 1. D 9. B 14. K 49. B 31. A
12. J 2. M 10. J 15. B 50. L 32. L
13. D 3. D 11. C 16. K 51. B 33. B
14. M 4. L 12. M 17. C 34. J
5. B 13. B 18. M Spelling 35. D
Antonyms 6. L 14. K 19. B 1. B 36. L
1. C 7. C 20. K 2. J 37. D
2. M Capitalization 21. C 3. C 38. J
3. B Sentence and 22. K 4. K
4. M Structure and Punctuation 23. D 5. B LatinWord
5. D Usage 1. C 24. J 6. J Roots
6. J 1. C 2. K 25. D 7. B 1. C
7. C 2. K 3. C 26. K 8. L 2. M
8. J 3. B 4. L 27. D 9. A 3. A
9. B 4. L 5. B 28. L 10. J 4. M
10. L 5. A 6. K 29. C 11. C 5. C
11. D 6. L 7. B 30. L 12. K 6. J
12. K 7. B 8. L 31. B 13. A 7. D
13. D 8. K 9. B 32. K 14. K 8. L
14. J 9. A 33. A 15. B 9. D
Reading 34. M 16. K 10. L
Context Comprehension 35. D 17. D 11. D
1. C 1. C 36. J 18. J 12. M
2. J 2. L 37. B 19. B 13. B

4th Grade Edition 45


MATH
Practice Skill: CONCEPTS/WHOLE NUMBER
COMPUTATION
Objective: To evaluate the use of numbers in various
mathematical situations and compute them accurately.

Parent Tip: Help your child master his addition, subtraction, multiplication, and division
facts. Your child must have strategies to work with larger number concepts such as
doubling, estimating, using comfortable numbers, solving a simpler problem first, trial
and error, and using number sense. These strategies are discussed in the back of the
book.

Example:

A hot air balloon can lift 400 pounds. Steve weighs 110 pounds, Alex weighs 140
pounds, Josè weighs 95 pounds, and Billy weighs 76 pounds. What is their combined
weight?

ȅ A. 821
ȅ B. 521
ȅ C. 421
ȅ D. 321

The correct answer is “C”. Total the boys’ weights. How much the balloon can lift is
unnecessary for this problem.

Choose the best answer:

1. Find the difference between 43,726 and 68,002.

ȅ A. 14,276
ȅ B. 24,276
ȅ C. 70,128
ȅ D. 71,728

2. Mr. Jones collects sports cards. He has 14,115 baseball cards, 8,721 basketball cards,
and 487 football cards. How large is his collection?

ȅ J. 150,025
ȅ K. 32,233
ȅ L. 23,323
ȅ M. 17,706

4th Grade Edition 46


3. Which product is greatest?

ȅ A. 525 x 15
ȅ B. 515 x 20
ȅ C. 500 x 25
ȅ D. 5251 x 5

4. Find the quotient for 4356 ÷ 6.

ȅ J. 826
ȅ K. 726
ȅ L. 626
ȅ M. 526

5. Which equation is true?

ȅ A. 425 x 5 = 848 x 3
ȅ B. 424 x 6 = 708 x 3
ȅ C. 425 x 5 = 708 x 3
ȅ D. 424 x 6 = 848 x 3

6. Which number is the best estimate for the average of 360 x 150?

ȅ J. 5,000
ȅ K. 50,000
ȅ L. 6,500
ȅ M. 8,000

7. What is the difference between 24 and 100?

ȅ A. 76
ȅ B. 124
ȅ C. –76
ȅ D. –124

4th Grade Edition 47


Practice Skill: NUMBER SENSE AND NUMERATION

Objective: To evaluate place value skills from hundredths to


millions. To evaluate an understanding of numbers by applying a
sense of reasonableness to a mathematical situation.

Parent Tip: Help your children master their understanding and use of place value. They
should be able to construct and use a place value chart from hundredths to millions.
Additionally, they must recognize the difference between whole number and decimal
values.

Example: Which statement is true?

ȅ A. 69.02 < 6.90


ȅ B. 6.90 > .690
ȅ C. 690. < 69.02
ȅ D. 690 > 690.2

Choose the best answer.

The correct answer is “B”. Six and nine-tenths is greater than sixty-nine hundredths.
Students must compare each of the number values and then the use of the >,<, or =
symbols. They also need to understand the placement and value of zero in whole number
and decimal locations.

1. Which set of numbers is listed from greatest to least?

ȅ A. (84,810 85,810 86,810 87,810)


ȅ B. (84,100 84,200 84,300 84,400)
ȅ C. (87,810 86,810 85,810 84,810)
ȅ D. (84,400 84,300 84,100 84,200)

2. Which number is represented by (6 x 1,000,000) + (4 x 10,000) + (6 x 1000) +


(5 x 100) + (6 x 10) + (5 x 1)?

ȅ J. 646,565
ȅ K. 640,565
ȅ L. 6,460,565
ȅ M. 6,046,565

4th Grade Edition 48


3. Which number has the digit 9 in the ten thousands place?

ȅ A. 6,909,432
ȅ B. 6,090,932
ȅ C. 6,900,932
ȅ D. 9,609,932

4. What is the sum of 864,421 + 1000?

ȅ J. 874,421
ȅ K. 974,421
ȅ L. 865,421
ȅ M. 865,521

5. Which value is the greatest?

ȅ A. 1012.16
ȅ B. 1012.06
ȅ C. 1021.06
ȅ D. 1021.26

6. Which answer is ordered from least to greatest?

ȅ J. 71.02, 7110.2, 7102.11


ȅ K. 111.02, 101.02, 102.01
ȅ L. 31.03, 310.30, 3103.03
ȅ M. 123.4, 102.34, 1234.12

4th Grade Edition 49


Practice Skill: MEASUREMENT AND GEOMETRY

Objective: To evaluate an understanding of plane and solid


geometric shapes by identifying and applying mathematical
formulas and solving problems.

Parent Tip: Help your child understand the difference between plane two-dimensional
objects and solid three-dimensional geometric objects (square vs. cube). She must be
familiar with the vocabulary of geometry such as: faces, edges, vertices, etc., listed in the
back of this book. Additionally, she must know the mathematical formulas for perimeter,
area, and volume, also listed in the back of this book.

Example: What is the area for the shaded space inside the figure below?

ȅ A. 24 units2
ȅ B. 17 units2
ȅ C. 20 units2
ȅ D. 19 units2

The correct answer is “B”. To get this, you count the total number of shaded squares
(14). Then count the shaded triangles (6). Two triangles equal one square unit.
So, 14 + 3 = 17

Choose the best answer:

1. Which figure is congruent to the first?

A. B. C. D.

4th Grade Edition 50


2. Which shape has a line of symmetry?
ȅ J. ȅ K. ȅ L.

3. Which figure has a measurement of 180°?

ȅ A. ȅ B. ȅ C.

4. Which figure has parallel lines?

ȅ J. ȅ K. ȅ L.

5. Which pair of lines are perpendicular?

ȅ A. ȅ B. ȅ C.

6. Which is the best estimate for the area of the following figure?
1.5’
1’
2’

6’

ȅ J. 12 feet2 ȅ K. 16 feet2 ȅ L. 8 feet2

4th Grade Edition 51


7. How long is a horizontal line segment with endpoints (-3,2) and (2,2)?

ȅ A. 1
ȅ B. 6
ȅ C. 5

8. What is the radius of a circle with a diameter of 14cm?

ȅ J. 7cm
ȅ K. 24cm
ȅ L. 28cm

9. How many faces does a rectangular prism have?

ȅ A. none
ȅ B. 12
ȅ C. 6

10. Which figure represents an equilateral triangle?

ȅ J. ȅ K. ȅ L.

11. A ____________ has 8 edges.

ȅ A. square
ȅ B. cube
ȅ C. pyramid

4th Grade Edition 52


Practice Skill: STATISTICS AND PROBABILITY

Objective: To evaluate interpretation of numerical data. To


evaluate the ability to make predictions for simple probability
situations.

Parent Tip: Help your child to interpret different types of graphs and charts. Help him
practice using mode, median, range, and average (formulas in glossary). Lastly, he needs
to be familiar with expressing outcomes of probability situations numerically (e.g. 3 out
of 4; ¾).

Example: At a recent bowling tournament, Steve Star scored 293, 284, 272, 279, and
212 in 5 games. What was his average score?

ȅ A. 279
ȅ B. 268
ȅ C. 272
ȅ D. 270

The correct answer is “B”. You must total the 5 scores (1340) and divide by the number
of scores (5). The result is 268.

Use the table below for questions 1 through 3.

Candy Store
Inventory
Name of Product Pieces per Box # Boxes per Package
A. Chocolate Raisins 121 12
B. Chocolate Peanuts 95 12
C. Cream Caramels 60 24
D. Luscious Lollys 75 10

1. Which package has the greatest number of total pieces?

ȅ A. Chocolate Raisins
ȅ B. Chocolate Peanuts
ȅ C. Cream Caramels
ȅ D. Luscious Lollys

4th Grade Edition 53


2. If all candy packages are priced equally, which package has the greatest cost per
piece?

ȅ J. Chocolate Raisins
ȅ K. Chocolate Peanuts
ȅ L. Cream Caramels
ȅ M. Luscious Lollys

3. How many boxes of Cream Caramels come in 5 packages?

ȅ A. 100
ȅ B. 1440
ȅ C. 120
ȅ D. 240

Use the following graph to answer questions 4 through 6.

A survey asked nuts


100 people to
name their cookie
favorite snack.
chips
Cheese
sticks
carrots

Granola bars

Match each response with the correct percent:

4. Cookies were chosen by:

ȅ J. 50 % ȅ K. 25 % ȅ L. 18 % ȅ M. 35 %

5. Carrots were chosen by:

ȅ A. 5 % ȅ B. 10 % ȅ C. 15 % ȅ D. 20 %

6. Nuts and/or cookies were chosen by:

ȅ J. 32 % ȅ K. 18 % ȅ L. 40 % ȅ M. 50 %

4th Grade Edition 54


Use the following diagram to answer questions 7 through 10.

purple blue

red black

Predict the results of 100 spins.

7. The spinner will land on purple.

ȅ A. 1/10
ȅ B. 50 %
ȅ C. 25/100
ȅ D. 100 %

8. The spinner will land on either purple or black.

ȅ J. 1/10
ȅ K. 50 %
ȅ L. 25/100
ȅ M. 100 %

9. Red has the same chance as:

ȅ A. purple
ȅ B. blue
ȅ C. black
ȅ D. all of the above

10. The spinner has a 3 in 4 chance to land on red, black, and ___________ combined.

ȅ J. purple
ȅ K. red
ȅ L. black
ȅ M. none of the above

4th Grade Edition 55


Practice Skill: FRACTION AND DECIMAL CONCEPTS

Objective: To evaluate the ability to interpret parts of a whole,


parts of a set, use division concepts, and relate to decimal values.

Parent Tip: Help children to develop fractional sense by constantly exposing them to
units cut or divided into equal parts. Help them see doubling patterns such as halves,
quarters, eighths, sixteenths, etc. When working with odd numbered denominators, use a
“log” to illustrate the value (such as a loaf of bread). Students too often become focused
on boxes and circles for creating fractions and become confused when they have to draw
fifths or sevenths. Additionally, they need to become aware of parts of groups. Decimal
concepts need to be identified as special fractions that are based on units of 10. Decimals
are fractional parts. Using money to create an understanding about hundredth and tenths
is most practical.

Example: Identify the portion that is not shaded.

ȅ A. 4/10 ȅ B. 2/5 ȅ C. 3/5 ȅ D. 50 %

The correct answer is “C”. Six out of 10 boxes are not shaded. The fraction 6/10 is
equivalent to 3/5 by reducing to lowest terms.

Choose the best answer.

1. A large pizza has 12 slices. 2/3 of the pizza has been eaten. How many slices have
been eaten?

ȅ A. 2
ȅ B. 3
ȅ C. 7
ȅ D. 8

4th Grade Edition 56


Use the diagram to answer questions 2 and 3.

2. Look inside the large rectangle. What fraction of the shapes are triangles?

ȅ J. 3/12
ȅ K. 4/4
ȅ L. 1/5
ȅ M. 1/3

3. What fraction of the shapes has straight lines?

ȅ A. 4/12
ȅ B. 1/5
ȅ C. 8/12
ȅ D. 1/3

4. Which decimal has the same value as ¾?

ȅ J. .34
ȅ K. .43
ȅ L. .75
ȅ M. .56

5. Which decimal has the same value as 125/100?

ȅ A. .125
ȅ B. 1.25
ȅ C. 12.5
ȅ D. 125.

4th Grade Edition 57


Practice Skill: PATTERNS AND RELATIONSHIPS

Objective: To evaluate the ability to analyze patterns and


relationships in order to predict the additional items in a series.

Parent Tip: Help your child to analyze the pattern before him. He should look for a
visual pattern and draw out what he thinks is next. Or, when using number patterns, he
should identify the mathematical equation within the pattern and then apply it.

Example: Fill in the blank.

ABBC, BCCD, _________, DEEF

ȅ A. CBBD ȅ B. CDDE ȅ C. DCCE ȅ D. EFFG

The correct answer is “B”. The first letter in each item is in alphabetical order, followed
by the doubling of the next alphabetical letter, and ending with a single alphabetically
ordered letter in the chain.

Choose the best answer.

1. Which will come next? 18, 36, 72, _____,

ȅ A. 108
ȅ B. 124
ȅ C. 144
ȅ D. 98

2. Which rule is correct for the following? 2, 8, 20, 44,

ȅ J. yx2+2
ȅ K. y+4x2
ȅ L. y+3x3
ȅ M. y+2x2

3. Which pattern is next?

ȅ A. ȅ B. ȅ C. ȅ D.

4. Which fraction does not belong in the pattern? 1/5, 2/10, 3/15, 4/25

ȅ J. 1/5 ȅ K. 2/10 ȅ L. 3/15 ȅ M. 4/25

4th Grade Edition 58


Practice Skill: ESTIMATION

Objective: To evaluate the ability to appropriately estimate


mathematical values and solve estimation.

Parent Tip: Help your children to use estimation skills on a regular basis. Estimation
skills help to develop a greater math sense. Students may use different estimation
strategies depending upon the type of math problem. Traditional rounding may be used
with very specific directions such as rounding to the nearest hundred. Larger estimation
skills are used in situations that call for an approximate or “about” type answer. Students
often use compatible or friendly numbers that make sense to solve a problem. This
actually simplifies the math.

Example: Mr. Smith bought 5 cans of dog food at 67¢ each. About how much did they
cost all together?

ȅ A. $3.00 ȅ B. 70¢ ȅ C. $3.50 ȅ D. $3.35

The correct answer is “C”. When the word “about” is used in a problem, you are being
directed to use estimation skills. 67¢ is closest to 70¢. (70¢ x 5 = $3.50)

1. Estimate the quotient for 278 ÷ 4 =

ȅ A. 60
ȅ B. 65
ȅ C. 70
ȅ D. 75

2. Estimate the product for 38 x 18 =

ȅ J. 400
ȅ K. 800
ȅ L. 300
ȅ M. 500

3. About how much will it cost to buy 3 pairs of shoes priced at $39.95, $24.95, and
$12.99?

ȅ A. $80.00 ȅ B. $70.00 ȅ C. $60.00 ȅ D. $50.00

4. About how much money should Mrs. Jackson plan to spend if she buys 10 azeala plants
at $2.95 each. Be sure she overestimates.

ȅ J. $25.00 ȅ K. $30.00 ȅ L. $35.00 ȅ M. $40.00

4th Grade Edition 59


Practice Skill: PROBLEM SOLVING STRATEGIES

Objective: To evaluate the ability to analyze a math problem and


apply an appropriate problem solving strategy.

Parent Tip: Help your children to solve word problems by getting them to read the
question CAREFULLY. They should write out an equation or draw a picture to
complement their thinking. When in doubt, draw it out! Creating a visual can be of
monumental assistance, especially to students who struggle in the language area.
Additionally, they need to work through a problem using their thinking style rather than
always having to follow traditional equations. The way math was taught to you 20 to 30
years ago is not the way that students have to solve math today.

Strong basic skills will help build success in problem solving. By the fourth grade, math
problems involve large numbers, and if students are struggling with the basic facts, they
may feel frustrated with multiple step problems.

Example:

Jenny is raising mice for a science experiment. She has a total of 36 mice in a
small and large cage. The large cage has double the number of mice. How many are in
the small cage?

ȅ A. 24 ȅ B. 36 ȅ C. 18 ȅ D. 12

The correct answer is “D”. The small cage has 12 while the large has 24. 24 is double
12 and their sum is 36.

1. Samantha had 3 types of meat: turkey, roast beef, and ham. She had 2 types of
bread: white and wheat. How many sandwich choices are there if there is only one
type of meat per sandwich?

ȅ A. 3
ȅ B. 6
ȅ C. 9
ȅ D. 12

2. Juanita enjoys bike riding. She rides at a rate of 2.5 miles every 10 minutes. If she
rides steadily at this rate how far will she go in a ½ hour?

ȅ J. 25 miles
ȅ K. 15 miles
ȅ L. 7.5 miles
ȅ M. 9.5 miles

4th Grade Edition 60


3. Kathy decided to give away her coin collection. She sold 124 coins, then equally
divided the rest among her 3 sisters. They each got 69. How many coins had been
in Kathy’s collection?

ȅ A. 331
ȅ B. 207
ȅ C. 173
ȅ D. 176

4. A 1 pound package of chocolate candies has 48 pieces. 1/6 of them are caramels,
1/3 have nuts, and ½ are either light or dark solids. How many have nuts?

ȅ J. 8
ȅ K. 16
ȅ L. 12
ȅ M. 20

5. At Public School #111, the milk is sold in ½ pint containers. On an average day,
the school sells 632 containers. How many gallons of milk is that?

ȅ A. 390
ȅ B. 39.5
ȅ C. 316
ȅ D. 158

4th Grade Edition 61


Practice Skill: LOGIC AND PROBLEM SOLVING

Objective: To evaluate knowledge of problem solving strategies


that involve pre-algebraic thinking and multiple-step procedures.

Parent Tip: Help your children to feel successful when approaching word problems by
having them draw or list parts of the problem. Many students have difficulty with these
types of problems because they are not the basis of many math programs. However, they
are just the type of problem that students will encounter in tests of math skills. Students
quickly become better at these problems as they try more and more of them. At times,
students can solve problems without traditional equations; however, the stronger their
basic math skills and math sense, the more likely they are to solve these problems.

Children should be familiar with such strategies as working backwards, solving a simpler
problem first, trial and error, and logical estimation. They also need to have a strong
math vocabulary to analyze and understand what some questions are asking.

Choose the best answer.

1. When a clock reads 10:15, what time would it read if you were looking into a mirror?

ȅ A. 10:15
ȅ B. 11:15
ȅ C. 1:45
ȅ D. 11:45

2. If you have twelve squares, you can arrange them to make three different types of
rectangular shapes. (For example: 12 x 1; 6 x 2; and 3 x 4). How many squares
would you need in order to make 4 different rectangular shapes?

ȅ J. 4
ȅ K. 16
ȅ L. 24
ȅ M. 36

3. Which of the following is a prime number?

ȅ A. 24
ȅ B. 57
ȅ C. 33
ȅ D. 21

4th Grade Edition 62


4. If everyone in a class of 20 students shakes hands with everyone else just once, how
many handshakes were there?

ȅ J. 19
ȅ K. 63
ȅ L. 190
ȅ M. 400

5. A chime on a clock strikes one chime at one o’clock, two chimes at two o’clock, up
to twelve chimes at twelve o’clock. What is the total number of chimes that a clock
will strike during a twelve-hour period?

ȅ A. 60
ȅ B. 66
ȅ C. 72
ȅ D. 78

6. Which two consecutive numbers will total 225?

ȅ J. 100 and 125


ȅ K. 110 and 115
ȅ L. 112 and 113
ȅ M. 120 and 105

7. How many cubes are necessary to build the figure below, if the figure is solid?

ȅ A. 24
ȅ B. 48
ȅ C. 60
ȅ D. 72

8. Read the clues and find the number:

The number has three digits.


The tens digit is one-half the hundreds digit.
The number is odd.
The sum of the digits is nine.

ȅ J. 243
ȅ K. 216
ȅ L. 621
ȅ M. 423

4th Grade Edition 63


9. Arrange the digits 2, 4, 7, and 9 so that you get the largest possible product.

ȅ A. 97 x 24
ȅ B. 94 x 72
ȅ C. 92 x 74
ȅ D. 79 x 42

10. If you start with one whole and you cut the pieces in half repeatedly three times, how
much do you have?

ȅ J. two halves
ȅ K. three thirds
ȅ L. four quarters
ȅ M. eight eighths

11. Suzanne has six times as many marbles as Kathy. John has half as many as Judy.
Judy has half as many as Suzanne. Kathy has four marbles. How many marbles do
John and Suzanne have together?

ȅ A. 16
ȅ B. 24
ȅ C. 30
ȅ D. 36

12. The sum of the digits of an odd two-digit prime number is 11. The tens digit is
greater than the ones digit. What is the number?

ȅ J. 92
ȅ K. 47
ȅ L. 83
ȅ M. 38

13. A baker is getting ready for the day. He is mixing a very large amount of batter to be
able to make several cakes throughout the day. If he splits the batter once, he now
has two batches. If he splits those two batches he will then have four. If he continues
this process four more times, how many batches of batter will he have?

ȅ A. 6
ȅ B. 16
ȅ C. 30
ȅ D. 64

4th Grade Edition 64


14. Freddie keeps all his socks in one drawer. He has seven blue socks and nine brown
socks. If he reaches in the drawer without looking, what is the fewest number of
socks he can take out to be sure of getting a pair of the same color?

ȅ J. 2
ȅ K. 3
ȅ L. 5
ȅ M. 16

15. If it is 10:00 p.m. in California, what time is it in New York City?

ȅ A. 7:00 p.m.
ȅ B. 1:00 a.m.
ȅ C. 1:00 p.m.
ȅ D. 7:00 a.m.

16. A farmer had 457 milk cows. He lost all but 112 of them to disease. How many were
left alive?

ȅ J. 369
ȅ K. 112
ȅ L. 569
ȅ M. 445

17. Andy’s plant and garden shop sells four rose bushes for every three gardenias. Last
month, they sold 48 roses bushes. How many gardenias were sold?

ȅ A. 48
ȅ B. 18
ȅ C. 12
ȅ D. 36

18. Mrs. Johnson had baked ¾ of the cookies she needed for the bake sale. What
percentage of the cookies had she baked?

ȅ J. 20 %
ȅ K. 25 %
ȅ L. 65 %
ȅ M. 75 %

19. Complete the pattern: 1, 1, 2, 3, 5, 8, ___, ___, ___

ȅ A. 15, 17, 19
ȅ B. 12, 17, 23
ȅ C. 13, 21, 34
ȅ D. 13, 22, 35

4th Grade Edition 65


Practice Skill: MATH COMPUTATION

Objective: To evaluate computation skills using the four


operations, fractions, decimals, and percentages.

Parent Tip: To be successful in basic computation, your child must know her addition,
subtraction, multiplication, and division facts with complete accuracy. Most errors are
the result of calculation mistakes based on inadequate knowledge of facts. Review the
procedures for large multiplication, long division, adding and subtracting fractions,
working with decimals, and percentages. Repeated practice helps improve accuracy.
Additionally, encourage your child to develop her “math sense.” This means to evaluate
an answer to determine if the answer is logical for the problem. Try to have your child
consistently compare her answer back to the original question. Estimation skills are also
highly valuable as well.

Students also need to know the following terms:

sum difference
product quotient
decimal value fractional value
mixed number improper fraction
simplest terms equivalent fractions
average percentage

The math glossary for this unit will provide resource information for required vocabulary.

Solve the following and choose the best answer.

1. Find the product for 432 x 28.

ȅ A. 4320
ȅ B. 4520
ȅ C. 12,096
ȅ D. 11,096

2. What is the difference for 18,642 and 12, 946?

ȅ J. 5696
ȅ K. 5606
ȅ L. 6696
ȅ M. 6606

4th Grade Edition 66


3. Find the sum for 836 + 12,666 + 42 =

ȅ A. 100,466
ȅ B. 98,042
ȅ C. 13,544
ȅ D. 12,534

4. Find the quotient for 836 divided by 9.

ȅ J. 90 r.9
ȅ K. 91 r.9
ȅ L. 92 r.8
ȅ M. 93 r.8

5. Which equation shows calculating an average?

ȅ A. 98(76 + 89 + 44) x 4 =
ȅ B. (98 + 76 + 89 + 44) ÷ 4 =
ȅ C. (98 + 76) ÷ (89 + 44) =
ȅ D. (98 + 76 + 89 + 44) x 4 =

6. Solve 5672 ÷ 24 =

ȅ J. 226 r.10
ȅ K. 236 r.8
ȅ L. 246 r.10
ȅ M. 256 r.8

7. How much is 25 % of 4?

ȅ A. 1
ȅ B. 4
ȅ C. 8
ȅ D. 16

8. 75 % of 800 =

ȅ J. 800
ȅ K. 700
ȅ L. 600
ȅ M. 500

4th Grade Edition 67


9. If you score 18/25 on a test your grade would be:

ȅ A. 90 %
ȅ B. 82 %
ȅ C. 80 %
ȅ D. 72 %

10. Solve and name in simplest terms: 5/9 + 6/9 =

ȅ J. 1 1/9
ȅ K. 1 1/5
ȅ L. 1 1/3
ȅ M. 1 2/9

11. Solve and name in simplest terms: 12/15 – 1/3 =

ȅ A. 11/12
ȅ B. 17/15
ȅ C. 7/15
ȅ D. 13/18

12. Complete the pattern: 2/6, 5/15, 9/27, ____

ȅ J. 9/27
ȅ K. 10/33
ȅ L. 11/22
ȅ M. 12/36

13. Solve 450 x 0.5 =

ȅ A. 225
ȅ B. 900
ȅ C. 2250
ȅ D. 1250

14. At the store Sam buys three calculators for $2.99 each. There is no tax. If he pays
with a twenty-dollar bill, how much change will he receive?

ȅ J. (8) $1.00 bills, (1) $5.00 bill, 2 quarters, and 3 pennies


ȅ K. (2) $5.00 bills, (1) $1.00 bill, and 3 pennies
ȅ L. (1) $10.00 bill, (3) $1.00 bills, and 1 penny
ȅ M. (11) $1.00 bills, and 11 pennies

4th Grade Edition 68


15. Solve 1215 ÷ ______ = 135

ȅ A. 135
ȅ B. 7
ȅ C. 8
ȅ D. 9

16. Solve 456 x ______ = 5472

ȅ J. 10
ȅ K. 11
ȅ L. 12
ȅ M. 13

17. Use compatible numbers to estimate 42 x 311 =

ȅ A. 120
ȅ B. 1200
ȅ C. 12,000
ȅ D. 120,000

18. ½ x ½ = is the same as

ȅ J. ½
ȅ K. ¼
ȅ L. 1/6
ȅ M. 1/8

19. ¼ of 300 =

ȅ A. 110
ȅ B. 100
ȅ C. 95
ȅ D. 75

20. Mrs. Sylvester went shopping at Mark’s Department Store and spent $112.50. She
had to pay 10 % sales tax as well. Her total bill was:

ȅ J. $123.75
ȅ K. $135.50
ȅ L. $124.50
ȅ M. $124.00

4th Grade Edition 69


MATH
Answer Key

Concepts/Whol 6. M 5. D
e Number 7. C 6. L
Computation 8. K 7. B
1. B 9. D 8. M
2. L 10. J 9. C
3. D 10. M
4. K Fraction and 11. C
5. D Decimal Concepts 12. L
6. K 1. D 13. D
7. A 2. M 14. K
3. C 15. B
Number Sense 4. L 16. K
and Numeration 5. B 17. D
1. C 18. M
2. M Patterns and 19. C
3. B Relationships
4. L 1. C Math
5. D 2. M Computation
6. L 3. B 1. C
4. M 2. J
Measurement and 3. C
Geometry Estimation 4. L
1. C 1. C 5. B
2. K 2. K 6. K
3. C 3. A 7. A
4. K 4. K 8. L
5. A 9. D
6. L Problem Solving 10. M
7. C Strategies 11. C
8. J 1. B 12. M
9. C 2. L 13. A
10. K 3. A 14. K
11. B 4. K 15. D
5. B 16. L
Statistics and 17. C
Probability Logic and 18. K
1. A Problem Solving 19. D
2. M 1. C 20. J
3. C 2. L
4. L 3. B
5. A 4. L

4th Grade Edition 70


SOCIAL STUDIES

Practice Skill: GEOGRAPHY

Objective: To evaluate knowledge and use of general map skills,


knowledge of world geography and the effects of geography on
society.

Parent Tip: Help your children to practice reading maps, charts, and pictures related to
geography. They should be able to use a variety of grid systems, particularly latitude and
longitude. Their basic sense of world geography should include the continents, four
hemispheres, the poles, and major land features. Additionally, students should know the
geographic regions and land features of the United States, and understand their impact on
the economic development of the states in each region.

Choose the best answer.

1. The line of latitude that divides the globe into two equal hemispheres is the

ȅ A. prime meridian.
ȅ B. Greenwich meridian.
ȅ C. equator.
ȅ D. primary locator.

2. The vertical lines on the globe that run north to south

ȅ J. measure distance from north to south.


ȅ K. measure distance from east to west.
ȅ L. go around the globe without intersecting the poles.
ȅ M. begin at the equator to divide the northern hemisphere and the southern
hemisphere.

3. Which sentence is true?

ȅ A. To travel from Boston to Orlando, you would go east.


ȅ B. To travel from Arizona to Illinois, you would go west.
ȅ C. To travel from Mississippi to Michigan, you would go north.
ȅ D. To travel from California to Oregon, you would go south.

4th Grade Edition 71


4. A topographical map will give details about
ȅ J. the plants and animals that live in a region.
ȅ K. highways, street names and points of interest.
ȅ L. the weather in a region.
ȅ M. the shape of the land and elevations.

Use the world map to identify the locations in exercises 5 - 9 :

5 8
1

3 4
2 6
7
9

5. North America
ȅ A. 1
ȅ B. 2
ȅ C. 4
ȅ D. 7

6. Mexico 8. Pacific Ocean


ȅ J. 1 ȅ J. 3
ȅ K. 2 ȅ K. 5
ȅ L. 4 ȅ L. 6
ȅ M. 7 ȅ M. 9

7. Europe 9. Panama Canal

ȅ A. 3 ȅ A. 2
ȅ B. 5 ȅ B. 5
ȅ C. 6 ȅ C. 7
ȅ D. 8 ȅ D. 9

4th Grade Edition 72


10. Which of the following are reasons that a city might be founded near a river?

ȅ J. The river water is difficult to pollute.


ȅ K. There are no disadvantages to living near a river.
ȅ L. The river may be used as a transportation route and a source of food.
ȅ M. All of the above

11. Water, soil, trees, gold, oil and fish are examples of

ȅ A. natural hazards.
ȅ B. technology.
ȅ C. natural resources.
ȅ D. artifacts.

12. The source of the Mississippi River is in

ȅ J. Louisiana.
ȅ K. Gulf of Mexico.
ȅ L. Mississippi.
ȅ M. Minnesota.

13. Which of the world’s oldest mountain ranges runs through the Northeast and the
Southeast of the United States?

ȅ A. Sierra Nevada
ȅ B. Rocky Mountains
ȅ C. Grand Tetons
ȅ D. Appalachian Mountains

14. The major landform in the United States that is located between the Appalachian
and Rocky Mountains is

ȅ J. the Interior Plains, which are made up of the Great Plans and the Central
Plains.
ȅ K. the Sonora Desert.
ȅ L. the wetlands of the Okefenokee Swamp and the Florida Everglades.
ȅ M. the Great Central Valley.

15. A student who wants to research current geographic, population, and economic
data for any location in the United States should consult

ȅ A. the U.S. Weather Service web site.


ȅ B. the U.S. Census Bureau web site.
ȅ C. the U.S. Environmental Protection Agency web site.
ȅ D. the U.S. Geological Survey map.

4th Grade Edition 73


Practice Skill: ECONOMICS
Objective: To evaluate knowledge and understanding of basic
economic concepts.

Parent Tip: Help your children to compare the kinds of goods and services that have
been produced in the past with goods and services that are important to their lives in the
twenty-first century. Point out to them the important goods and services provided by
federal, state and local governments by giving examples of how tax revenues are used.
The economic concepts of limited resources, production, and consumption are faced by
individuals and families in their daily lives in the same way that businesses and
government must address complex economic choices.

Choose the best answer.


1. Maria Galvan is an officer with the city police department. She patrols the
community streets on her bicycle or in a patrol car. Officer Galvan provides a
ȅ A. good.
ȅ B. service.
ȅ C. natural resource.
ȅ D. technology.

2. Someone who supplies goods and services to others is a


ȅ J. producer.
ȅ K. consumer.
ȅ L. demand changer.
ȅ M. exchanger.

3. A(n) ______________ is a person who takes a financial risk to start a business.


ȅ A. profit
ȅ B. trader
ȅ C. entrepreneur
ȅ D inventor

4. State governments get money to provide services to their residents by


ȅ J. selling gifts they receive from visiting foreigners.
ȅ K. keeping part of the money they print.
ȅ L. collecting taxes from families and businesses.
ȅ M. asking for donations from charities.

4th Grade Edition 74


5. Another word for income is
ȅ A. tax.
ȅ B. loan.
ȅ C. savings.
ȅ D. revenue.

6. An example of a private good or service is a

ȅ J. local restaurant.
ȅ K. fire department.
ȅ L. U.S. post office.
ȅ M. subway system.

7. Making many of the same things is called

ȅ A. limited resources.
ȅ B. free enterprise.
ȅ C. mass production.
ȅ D. product research and development.

8. Which of the following people is a producer?

ȅ J. Mrs. Jackson is building tool shed.


ȅ K. Dr. Olden is on the telephone ordering a pizza for dinner.
ȅ L. Shauna is visiting the zoo.
ȅ M. Gerald is going to the movies.

9. Which statement about the production of goods and services is not true?

ȅ A. Entrepreneurs are not important to the production of goods and services.


ȅ B. We use natural resources whenever we produce things.
ȅ C. Labor consists of the talents, skills and training of people that contribute to
the production of things.
ȅ D. Capital goods are used in production.

10. Which of the following are capital goods?

ȅ J. machines
ȅ K. vehicles
ȅ L. buildings
ȅ M. all of the above

4th Grade Edition 75


Practice Skill: CIVICS AND GOVERNMENT

Objective: To evaluate knowledge and understanding of ideals,


rights and responsibilities of American citizenship, and how the
government functions at various levels.

Parent Tip: An understanding of how to fully participate in our democracy is central to


social studies in grade 4. Students explore how they can make a positive difference in
their community and consider the roles of citizens at the state and national level. They
begin to examine how the meaning of citizenship has evolved in American society
through their study of history.
Help your children to understand the three branches of government and their
individual roles as well as their integrated roles in a system of checks and balances. Be
sure your children are aware of the differences between local, state, and national
governments and that the term “federal” refers to the national government. Discuss the
types of services that governments provide, and the ways that government policies and
actions affect our daily lives.

Choose the best answer.

1. The U.S. Constitution provides

ȅ A. a description for three branches of government at different (local, state, and


national) levels.
ȅ B. a letter from Thomas Jefferson about the role of government.
ȅ C. all levels of government with equal and same powers.
ȅ D. a structure for business opportunities.

2. Which branch of government is responsible for making laws?

ȅ J. Executive
ȅ K. Judicial
ȅ L. Legislative
ȅ M. Historical

3. Which branch of government is responsible for interpreting laws?

ȅ A. Executive
ȅ B. Judicial
ȅ C. Legislative
ȅ D. Historical

4th Grade Edition 76


4. Which branch of government is responsible for seeing that laws are carried out?
ȅ J. Executive
ȅ K. Judicial
ȅ L. Legislative
ȅ M. Historical

5. The term “federal” refers to government at the _________________ level.


ȅ A. district
ȅ B. local
ȅ C. state
ȅ D. national

6. A system of “checks and balances” refers to


ȅ J. a banking or budgeting method.
ȅ K. three branches of laws.
ȅ L. three branches of government.
ȅ M. a way to weigh or measure the importance of a law.

7. The leader of the federal government is the


ȅ A. President.
ȅ B. Governor.
ȅ C. Mayor.
ȅ D. City Council.

8. The leader of state government is the


ȅ J. President.
ȅ K. Governor.
ȅ L. Mayor.
ȅ M. City Council.

9. State governments are given power to


ȅ A. legislate laws related to the environment in their own state.
ȅ B. overrule federal laws.
ȅ C. print their own money.
ȅ D. change federal laws.

10. State governments are denied the power to


ȅ J. create laws to protect their environment.
ȅ K. declare war and keep their own armies.
ȅ L. establish traffic laws.
ȅ M. establish recycling programs.

4th Grade Edition 77


11. The federal government is similar to state governments in that they both

ȅ A. have three branches of government.


ȅ B. create new laws.
ȅ C. interpret laws.
ȅ D. All of the above.

12. One of the state government jobs is to control

ȅ J. treaties with foreign countries.


ȅ K. pollution.
ȅ L. printing of money.
ȅ M. taxation of imports.

13. Each state elects __________ United States Senators.

ȅ A. two
ȅ B. four
ȅ C. six
ȅ D. eight

14. It is the responsibility of the federal government to

ȅ J. feed all people.


ȅ K. clothe all people.
ȅ L. educate all people.
ȅ M. defend all people from attack by other countries.

15. The President of the United States is the leader of the nation’s

ȅ A. environmental protection group.


ȅ B. congressional committees.
ȅ C. military forces.
ȅ D. national choir.

16. An example of a citizen’s responsibility is to

ȅ J. make money.
ȅ K. have many friends.
ȅ L. travel across the country.
ȅ M. vote in every election.

4th Grade Edition 78


Practice Skill: HISTORY

Objective: To evaluate knowledge and understanding of the


relationship of people and events through time and significant
events in American history.

Parent Tip: Students in grade four study how democratic values came to be, and how
they are reflected in the people, historical events and symbols of America. They examine
family life and communities of long ago and compare them to the present. They gain
understanding of the contributions of people from other cultures who have influenced our
cultural, economic, and political heritage.

Choose the best answer.

1. When historians refer to life in the 17th century, they are referring to the years

ȅ A. 1700 – 1799.
ȅ B. 1700 – 1750.
ȅ C. 1600 – 1699.
ȅ D. 1800 – 1899.

2. Which of the following indicates a change in culture?

ȅ J. erosion of a hillside
ȅ K. buying a new car
ȅ L. electing a new president
ȅ M. millions of people using computers to work from their homes

3. The way we live has changed over time because of differences in

ȅ A. attitudes about hair styles and music.


ȅ B. communication and transportation.
ȅ C. cloud formations and the number of natural disasters that occur.
ȅ D. animal habits in the rivers and forests.

4. Some of the Native Americans who hunted, gathered food, and fished in the
woodlands, rivers, and oceans of the land east of the Mississippi River were the
ȅ J. Mohawk, Iroquois, Huron, and Winnebago.
ȅ K. Mandan, Sioux, Shoshone, Cheyenne, and Nez Perce.
ȅ L. Navajo, Hopi, and Zuni.
ȅ M. Chinook, Tlingit, Tillamook, and Makah.

4th Grade Edition 79


5. Which of the following would not be a part of the Eastern Woodlands culture?
ȅ A. pueblos
ȅ B. wampum
ȅ C. long houses
ȅ D. wigwams
6. Among the Native Americans of the Great Plains culture who followed the
buffalo herds for food were the
ȅ J. Mohawk, Iroquois, Huron, and Winnebago
ȅ K. Mandan, Sioux, Blackfoot, Crow, and Cheyenne
ȅ L. Navajo, Hopi, and Zuni
ȅ M. Chinook, Tlingit, and Tillamook
7. The Great Plains culture needed buffalo
ȅ A. meat for food.
ȅ B. hide for clothing and teepees.
ȅ C. bones for tools and weapons.
ȅ D. All of the above

8. In the Southwest, the Anasazi, also called the Basketmakers, lived in cliff cities
and were the ancestors of the
ȅ J. Mohawk, Iroquois, Huron, and Winnebago.
ȅ K. Mandan, Sioux, Shoshone, and Nez Perce.
ȅ L. Pueblo peoples, Hopi, and Zuni.
ȅ M. Chinook, Tlingit, and Inuit.

9. Which of the following would not be associated with various tribes of the
Southwest culture?

ȅ A. pueblos, hogans
ȅ B. Kachinas
ȅ C. dry farming
ȅ D. totem poles, potlatch ceremonies

10. In the Northwest, the _____________ lived in forests and traveled and fished the
lakes and rivers.
ȅ J. Mohawk, Iroquois, Huron, and Winnebago
ȅ K. Mandan, Sioux, Shoshone, and Nez Perce
ȅ L. Navajo, Hopi, and Zuni
ȅ M. Chinook, Tlingit, Tillamook, and Makah

4th Grade Edition 80


11. Which of the following would not be associated with the Pacific Northwest
culture?

ȅ A. totem poles and potlatch ceremonies


ȅ B. salmon fishing and whaling
ȅ C. pottery
ȅ D. wooden houses, baskets, and boxes

12. The Native Peoples who were considered part of the California culture and Great
Basin (between California and the Rocky Mountains) culture included the

ȅ J, Mohawk, Iroquois, Huron, and Winnebago.


ȅ K. Chumash, Yurok, Pomo, Paiute, and Nez Perce.
ȅ L. Navajo, Hopi, and Zuni.
ȅ M. Chinook Tlingit, Tillamook, and Makah.

13. Which of the following would not be associated with the California and Great
Basin cultures?

ȅ A. basket weaving
ȅ B. salmon fishing and whaling
ȅ C. hunting, fishing, and gathering seeds, nuts, berries, and tubers
ȅ D. round frame houses covered with grass, branches, or reeds

14. The first Europeans to gain a stronghold in the New World were from

ȅ J. England
ȅ K. Portugal
ȅ L. Spain
ȅ M. France

15. In 1519, Hernan Cortés

ȅ A. took his army through Mexico and seized the country.


ȅ B. defeated the Aztec emperor Montezuma.
ȅ C. captured all the wealth of the Aztec empire.
ȅ D. All of the above

16. The Spanish conquistadors

ȅ J. extensively explored Canada via the St. Lawrence River


ȅ K. explored California, but no other parts of North America
ȅ L. explored Peru, Mexico, Florida, California, Texas, and the American Southwest.
ȅ M. did not clash with the Native American peoples.

4th Grade Edition 81


17. Samuel de Champlain

ȅ A. was a Spanish conquistador


ȅ B. established Quebec and other settlements for France in Canada and America.
ȅ C. established fur trading and made friends with the Algonquin and Huron Indians.
ȅ D. Choices B and C.

18. Jacques Marquette and Louis Joliet

ȅ J. were French explorers who traveled the Mississippi River


ȅ K. claimed the Louisiana territory for King Louis XIV of France.
ȅ L. claimed for France all the land between the Appalachian Mountains and the
Rocky Mountains
ȅ M. All the above.

19. In 1609, Englishman Henry Hudson, sailing for ___________, established a claim
for the land around present day New York.

ȅ A. Holland
ȅ B. Spain
ȅ C. England
ȅ D. Spain

20. England’s claims to land in the New World were based on the explorations of

ȅ J. Christopher Columbus and Amerigo Vespucci.


ȅ K. Vasco de Balboa and Ferdinand Magellan.
ȅ L. John Cabot and Sir Francis Drake.
ȅ M. Henry Hudson and Jacques Cartier.

21. In 1607, the Indians in Virginia

ȅ A. killed all the colonists.


ȅ B. kept the Jamestown colonists from starving and taught them to grow tobacco.
ȅ C. fought with Powhatan from Canada.
ȅ D. None of the above

22. The most powerful organization of the Indian nations was

ȅ J. the Chumash in California.


ȅ K. Powhatan’s Confederacy of 30 tribes in the area of Virginia.
ȅ L. the Five Nations of the Iroquois with its Great Council.
ȅ M. the Hopi in Arizona.

4th Grade Edition 82


23. Which of the following statements is not true?
ȅ A. The Pilgrims were known in England as Puritans or Separatists.
ȅ B. The Puritans were part of a movement to reform the Church of England.
ȅ C. The Pilgrims wrote the Mayflower Compact and agreed to abide by laws created
for the good of the colony.
ȅ D. The Pilgrims established a colony in Roanoke, Virginia.

24. Which of the following statements is not true about England’s colonies in the
New World?
ȅ J. Rhode Island was founded to ensure religious freedom for its residents.
ȅ K. The Puritans were very tolerant of others’ religious beliefs.
ȅ L. Pennsylvania was established by Quaker William Penn.
ȅ M. African slaves often were brought to the colonies to replace indentured servants
who completed their work terms.

25. The first battles of the American Revolution were fought in

ȅ A. Lexington and Concord, Massachusetts.


ȅ B. Valley Forge, Pennsylvania.
ȅ C. Williamsburg, Virginia.
ȅ D. Boston harbor.

26. The American Revolution was a war

ȅ J. fought between France and the Netherlands over the Hudson River.
ȅ K. fought with England to create a new country that would not be ruled by the
king.
ȅ L. fought with Spain over the Mississippi River.
ȅ M. that the colonists fought without any help from France or Germany.

27. The rules for the government of the United States of America and the specific
rights of citizens are written in

ȅ A. the Farmer’s Almanac


ȅ B. the Declaration of Independence
ȅ C. the Constitution and the Bill of Rights
ȅ D. the Mayflower Compact

28. The American flag has fifty stars because

ȅ J. the Colonists had suffered for fifty years.


ȅ K. fifty people signed the Declaration of Independence.
ȅ L. each star represents a state.
ȅ M. there were fifty ships in the Navy.

4th Grade Edition 83


ANSWER KEY
SOCIAL STUDIES
Geography 13. A
1. C. 14. M
2. K 15. C
3. C 16. M
4. M
5. A History
6. L 1. C
7. B 2. M
8. J 3. B
9. A 4. J
10. L 5. A
11. C 6. K
12. M 7. D
13. D 8. L
14. J 9. D
15. B 10. M
11. C
Economics 12. K
1. B 13. B
2. J 14. L
3. C 15. D
4. L 16. L
5. D 17. D
6. J 18. M
7. C 19. A
8. J 20. K
9. A 21. B
10. M 22. L
23. D
Civics & Govt. 24. K
1. A 25. A
2. L. 26. K
3. B 27. C
4. J 28. L
5. D
6. L
7. A
8. K
9. A
10. K
11. D
12. K

4th Grade Edition 84


SCIENCE

Practice Skill: EARTH SCIENCE

Objective: To evaluate knowledge of basic geology concepts and


basic earth science principles.

Parent Tip: Help your children understand that minerals are the basic building blocks of
rocks. Rocks are one of the earth’s most common materials. Rocks are divided into three
sub-categories (igneous, metamorphic, and sedimentary). The study of rocks and
common earth materials provides information about the origin of our planet and its
ongoing development through time.
Additionally, other forces of nature cause great change in the earth, both short and long
term. Erosion and deposition constantly change the face of the planet during long
periods of time. Volcanoes and earthquakes can cause great change in short
periods of time.

Choose the best answer:

1. The basic building blocks of rocks are _____________________.

ȅ A. graphite
ȅ B. lava
ȅ C. minerals
ȅ D. jewelry

2. A property of a mineral is _____________________.

ȅ J. smoothness
ȅ K. hardness
ȅ L. color
ȅ M. All of the above.

3. __________________ is the softest of all minerals.

ȅ A. Talc
ȅ B. Asbestos
ȅ C. Graphite
ȅ D. Diamond

4th Grade Edition 85


4. Weathering is a process that _________________ rocks.

ȅ J. makes
ȅ K. erodes
ȅ L. melts
ȅ M. creates

5. Water is a(n) ____________________ that alters rocks.

ȅ A. actor
ȅ B. agent
ȅ C. mineral
ȅ D. living thing

6. Wind and ice will change rocks through _________________.

ȅ J. chemical change
ȅ K. seasonal change
ȅ L. physical change
ȅ M. heat

7. Rocks from the _______________ are most often changed by wind.

ȅ A. desert
ȅ B. beach
ȅ C. mountains
ȅ D. valleys

8. In a stream, as rocks hit each other they become ___________________.

ȅ J. broken
ȅ K. larger
ȅ L. round and smooth
ȅ M. sharp

9. ____________ is/are made from tiny bits of rock and the remains of living things.

ȅ A. Plants
ȅ B. Sand
ȅ C. Soil
ȅ D. Beaches

4th Grade Edition 86


10. _________________ rocks form as melted rocks.

ȅ J. Sedimentary
ȅ K. Igneous
ȅ L. Metamorphic
ȅ M. Seashore

11. As pieces of rock stick together and build, a(n) ____________ rock is being formed.

ȅ A. sedimentary
ȅ B. igneous
ȅ C. metamorphic
ȅ D. seashore

12. Heat and pressure change rocks into ________________ rocks.

ȅ J. sedimentary
ȅ K. igneous
ȅ L. metamorphic
ȅ M. seashore

13. Lava is an example of a(n) ________________ rock.

ȅ A. sedimentary
ȅ B. igneous
ȅ C. metamorphic
ȅ D. seashore

14. Changes in rocks due to heating, cooling, erosion, and pressure is known as

ȅ J. the water cycle.


ȅ K. the rock cycle.
ȅ L. the rubber cycle.
ȅ M. the oxygen cycle.

15. Limestone is changed to marble through applied pressure and extreme heat. Marble
is a(n) ______________ rock.

ȅ A. sedimentary
ȅ B. igneous
ȅ C. metamorphic
ȅ D. seashore

4th Grade Edition 87


16. An igneous rock can change into a sedimentary rock if

ȅ J. the rock is buried.


ȅ K. the rock is heated.
ȅ L. the rock is cooled
ȅ M. the rock is weathered.

17. Quartz, mica, and feldspar are all examples of

ȅ A. rocks.
ȅ B. minerals.
ȅ C. sands.
ȅ D. pebbles.

18. Minerals are made from molecules that form ____________________.

ȅ J. lumps
ȅ K. spheres
ȅ L. blocks
ȅ M. crystals

19. An example of a material that is soft is ___________________.

ȅ A. marble
ȅ B. chalk
ȅ C. granite
ȅ D. lead

20. Pollution is a form of __________________.

ȅ J. mineral
ȅ K. weathering
ȅ L. climate
ȅ M. property

21. Obsidian is a well-known ____________________ rock.

ȅ A. sedimentary
ȅ B. igneous
ȅ C. metamorphic
ȅ D. mountain

4th Grade Edition 88


22. Limestone is a well-known __________________rock.

ȅ J. sedimentary
ȅ K. igneous
ȅ L. metamorphic
ȅ M. mountain

23. Slate is a well-known _________________rock.

ȅ A. sedimentary
ȅ B. igneous
ȅ C. metamorphic
ȅ D. mountain

24. Heavy rains can cause serious ____________________.

ȅ J. erosion
ȅ K. deposition
ȅ L. slides
ȅ M. All of the above.

25. Scientists think that the continents may have been one super continent called ______.

ȅ A. Pangaea
ȅ B. tectonic plates
ȅ C. continental drift
ȅ D. continental plates

26. The most common cause of earthquakes is ___________________.

ȅ J. mountain movement
ȅ K. ocean movement
ȅ L. plate movement
ȅ M. lake movement

27. Common minerals found on the east coast of South America and the west coast of
Africa support the theory of ____________________.

ȅ A. continental drift
ȅ B. continental rift
ȅ C. continental shelf
ȅ D. continental ridge

4th Grade Edition 89


28. The ________________ of earth’s plates _____________ the height of mountains.

ȅ J. separating/decreases
ȅ K. shifting/decreases
ȅ L. shifting/increases
ȅ M. drifting/decreases

29. A ________________ is used to measure the strength of an earthquake.

ȅ A. television
ȅ B. sensorgraph
ȅ C. seismograph
ȅ D. tsunami

30. The ______________ fault is located in California.

ȅ J. Buenos Aires
ȅ K. San Andreas
ȅ L. Seismos
ȅ M. Continental

4th Grade Edition 90


Practice Skill: LIFE SCIENCE

Objective: To evaluate knowledge of the interactions between


plants and animals in the life cycle of the planet.

Parent Tip: Help your child understand the interactions of living and non-living things
on the Earth. She should be able to identify and analyze food chains and food webs,
common producers and consumers, and the distinction between omnivores, carnivores,
and herbivores. Additionally, she should be able to describe various ecosystems and
identify the members of their communities and their relationships.

Choose the correct answer.

1. An ecosystem is made up of ____________________.

ȅ A. living things
ȅ B. nonliving things
ȅ C. living and nonliving things
ȅ D. plants and animals

2. Living things sharing the same space is known as a

ȅ J. population.
ȅ K. community.
ȅ L. ecosystem.
ȅ M. habitat.

3. _______________ organisms are becoming reduced in number.

ȅ A. Thriving
ȅ B. Extinct
ȅ C. Endangered
ȅ D. Specialized

4. Organisms that live in the same place at the same time are an example of a

ȅ J. population.
ȅ K. community.
ȅ L. ecosystem.
ȅ M. competition.

4th Grade Edition 91


5. An organism’s _________________ is where it lives.

ȅ A. cave
ȅ B. plant
ȅ C. habitat
ȅ D. space

6. A ______________ is any living thing that cannot make its own food.

ȅ J. consumer
ȅ K. producer
ȅ L. decomposer
ȅ M. scavenger

7. A ______________ is a living thing that can make its own food.

ȅ A. consumer
ȅ B. producer
ȅ C. decomposer
ȅ D. scavenger

8. A ______________ is an animal that eats the remains of dead animals.

ȅ J. consumer
ȅ K. producer
ȅ L. decomposer
ȅ M. scavenger

9. A ______________ is an organism that breaks down other things to get food.

ȅ A. consumer
ȅ B. producer
ȅ C. decomposer
ȅ D. scavenger

10. An example of a ________________ is a worm.

ȅ J. consumer
ȅ K. producer
ȅ L. decomposer
ȅ M. scavenger

4th Grade Edition 92


11. An example of a _______________ is a vulture.

ȅ A. consumer
ȅ B. producer
ȅ C. decomposer
ȅ D. scavenger

12. An example of a _______________ is a fern.

ȅ J. consumer
ȅ K. producer
ȅ L. decomposer
ȅ M. scavenger

13. An example of a _______________ is a tucan.

ȅ A. consumer
ȅ B. producer
ȅ C. decomposer
ȅ D. scavenger

14. A diagram that shows how energy flows through an ecosystem

ȅ J. Caption
ȅ K. Food web
ȅ L. Food store
ȅ M. Story web

15. When organisms work together to help each other to survive, it is called

ȅ A. cooperation.
ȅ B. competition.
ȅ C. survival.
ȅ D. extinction.

16. When all of species die out, it is called

ȅ J. cooperation.
ȅ K. competition.
ȅ L. survival.
ȅ M. extinction.

4th Grade Edition 93


17. When organisms work against each other, it is called

ȅ A. cooperation.
ȅ B. competition.
ȅ C. survival.
ȅ D. extinction.

18. When organisms avoid competition they increase their chance for

ȅ J. cooperation.
ȅ K. competition.
ȅ L. survival.
ȅ M. extinction.

19. _________________ is an example of cooperation.

ȅ A. Competition
ȅ B. Niche
ȅ C. Symbiosis
ȅ D. Parasite

20. Fleas on a cat are an example of a

ȅ J. symbiotic relationship.
ȅ K. parasitic relationship.
ȅ L. friendly relationship.
ȅ M. community.

21. Animals that travel in herds use ___________ to increase their chances of ________.

ȅ A. fear/survival
ȅ B. competition/survival
ȅ C. cooperation/survival
ȅ D. energy/survival

22. An organism’s role in an ecosystem is its

ȅ J. title.
ȅ K. purpose.
ȅ L. prey.
ȅ M. niche.

4th Grade Edition 94


23. When a cat stalks a mouse, the cat is the ________ and the mouse is the _________.

ȅ A. prey/predator
ȅ B. predator/prey
ȅ C. competitor/prey
ȅ D. prey/competitor

24. Energy for all living things comes from the

ȅ J. earth.
ȅ K. moon.
ȅ L. planets.
ȅ M. sun.

25. As energy moves through a food chain it is

ȅ A. used by the first consumer.


ȅ B. passed on from one consumer to the next.
ȅ C. saved for the last consumer.
ȅ D. saved by producers.

26. An example of a likely food chain would be

ȅ J. plant – rodent – cat – sun.


ȅ K. sun – rodent – plant – cat.
ȅ L. cat – plant – rodent – sun.
ȅ M. sun – plant – rodent – cat.

27. An organism that eats only meat

ȅ A. carnivore
ȅ B. herbivore
ȅ C. omnivore
ȅ D. All of the above

28. An organism that eats plants and animals

ȅ J. carnivore
ȅ K. herbivore
ȅ L. omnivore
ȅ M. All of the above

4th Grade Edition 95


29. An organism that consumes plant matter

ȅ A. carnivore
ȅ B. herbivore
ȅ C. omnivore
ȅ D. All of the above

30. The Grizzly bear is an example of a(n)

ȅ J. carnivore.
ȅ K. herbivore.
ȅ L. omnivore.
ȅ M. All of the above

31. A butterfly is and example of a(n)

ȅ A. carnivore.
ȅ B. herbivore.
ȅ C. omnivore.
ȅ D. All of the above

32. A vulture is an example of a(n)

ȅ J. carnivore.
ȅ K. herbivore.
ȅ L. omnivore.
ȅ M. All of the above

4th Grade Edition 96


Practice Skill: PHYSICAL SCIENCE

Objective: To evaluate knowledge of energy systems such as


electricity and magnetism. To analyze electrical circuits, uses of
magnetism, and other power sources.

Parent Tip: Help your children understand that electricity and magnetism are examples
of the effects of electric charges. They should be familiar with the concept that the
gaining or losing of electrons results in objects becoming electrically charged.
Additionally, they should understand that magnets produce a “magnetic field” that creates
a force. Lastly, electrical circuits are pathways that move electric current, and different
materials conduct (move) electricity better than others.

Choose the correct answer.

1. The smallest parts of matter are _____________.

ȅ A. atoms
ȅ B. molecules
ȅ C. elements
ȅ D. electrons

2. The negative charge on an atom is called a(n) _____________.

ȅ J. atom
ȅ K. molecule
ȅ L. element
ȅ M. electron

3. As an object loses electrons, it will have a _____________ charge.

ȅ A. positive
ȅ B. negative
ȅ C. static
ȅ D. constant

4. When two objects attract each other, they are likely to have ___________ charges.

ȅ J. common
ȅ K. opposite
ȅ L. equal
ȅ M. static

4th Grade Edition 97


5. When two objects repel each other, they are likely to have ____________ charges.

ȅ A. common
ȅ B. opposite
ȅ C. electric
ȅ D. static

6. When there is an excess of non-moving electric charges in one place, you have
___________ charges.

ȅ J. common
ȅ K. opposite
ȅ L. equal
ȅ M. static

7. A ___________ is a particle of matter that carries a positive charge.

ȅ A. electron
ȅ B. proton
ȅ C. protron
ȅ D. neutron

8. ______________ are pathways for electricity to move through.

ȅ J. Conductors
ȅ K. Resistors
ȅ L. Transistors
ȅ M. Circuits

9. ______________ are materials that electrical charges move through easily.

ȅ A. Conductors
ȅ B. Resistors
ȅ C. Transistors
ȅ D. Circuits

10. _____________ are filaments found in light bulbs.

ȅ J. Conductors
ȅ K. Resistors
ȅ L. Transistors
ȅ M. Circuits

4th Grade Edition 98


11. Electricity is sometimes measured in _______________.

ȅ A. currents
ȅ B. volts
ȅ C. arcs
ȅ D. batteries

12. Electric cells can also be called __________________.

ȅ J. currents
ȅ K. volts
ȅ L. arcs
ȅ M. batteries

13. A _______________ is a switch that stops an overload of current.

ȅ A. amp
ȅ B. volt
ȅ C. circuit breaker
ȅ D. conductor

14. An example of a non-conductor is

ȅ J. copper.
ȅ K. silver.
ȅ L. aluminum.
ȅ M. rubber.

15. An example of a conductor is

ȅ A. aluminum.
ȅ B. wool.
ȅ C. cotton.
ȅ D. rubber.

16. A common light switch in the “off” position _______________________.

ȅ J. has electricity flowing


ȅ K. has a break in the circuit stopping the flow of electricity
ȅ L. has a broken switch
ȅ M. has an open circuit

4th Grade Edition 99


17. A fuse will _________ to stop the flow of electricity in an overload.

ȅ A. snap
ȅ B. quit
ȅ C. overload
ȅ D. melt

18. An unsafe situation is found when there is a(n) _________________.

ȅ J. open circuit
ȅ K. closed circuit
ȅ L. short circuit
ȅ M. long circuit

19. A magnet has ________ pole(s).

ȅ A. one
ȅ B. two
ȅ C. three
ȅ D. four

20. Magnetic poles are labeled _________ and _________.

ȅ J. top/bottom
ȅ K. weak/strong
ȅ L. south/north
ȅ M. west/east

21. When magnets pull toward each other they are said to _________________.

ȅ A. attract
ȅ B. repel
ȅ C. connect
ȅ D. disperse

22. When magnets push away from each other they are said to ______________.

ȅ J. attract
ȅ K. repel
ȅ L. connect
ȅ M. disperse

4th Grade Edition 100


23. True magnetic north is located at.

ȅ A. the northern hemisphere.


ȅ B. the North Pole.
ȅ C. the southern hemisphere.
ȅ D. the South Pole.

24. A core of iron or steel wrapped in a coil of electric current is

ȅ J. an example of current electricity.


ȅ K. an example of a hydroelectricity.
ȅ L. an example of an electromagnet.
ȅ M. an example of a short circuit.

25. The term AC stands for

ȅ A. active current.
ȅ B. ancient current.
ȅ C. always current.
ȅ D. alternating current.

26. The term DC stands for

ȅ J. designated current.
ȅ K. direct current.
ȅ L. dispersed current.
ȅ M. determined current.

27. A D-cell battery uses _______________ energy to produce electricity.

ȅ A. chemical
ȅ B. physical
ȅ C. recycled
ȅ D. constant

28. Electrical currents that flow in only one direction are examples of

ȅ J. alternating currents.
ȅ K. inconsistent currents.
ȅ L. consistent currents.
ȅ M. direct currents.

4th Grade Edition 101


29. Diagram “A” shows an example of a Diagram A

ȅ A. parallel circuit.
ȅ B. cereal circuit.
ȅ C. series circuit.
ȅ D. breaker circuit.

30. In diagram “A” what will happen when you remove a bulb?

ȅ J. All three bulbs will remain lit.


ȅ K. The two remaining bulbs will remain lit.
ȅ L. The two remaining bulbs will go out.
ȅ M. All bulbs are burned out.

31. Diagram “B” shows an example of a Diagram B

ȅ A. parallel circuit.
ȅ B. cereal circuit.
ȅ C. series circuit.
ȅ D. breaker circuit.

32. In diagram “B” what will happen if you remove a bulb?

ȅ J. All three bulbs will remain lit.


ȅ K. The two remaining bulbs will remain lit.
ȅ L. The two remaining bulbs will go out.
ȅ M. All bulbs are burned out.

33. Compare diagrams “A” with “B.” In which situation will the bulbs glow more
brightly?

ȅ A. diagram “A”
ȅ B. diagram “B”
ȅ C. both will be the same
ȅ D. all of the above

4th Grade Edition 102


34. In diagram “C,” which bulb(s) will light when the switch is closed?

Diagram C
ȅ J. A
ȅ K. B
ȅ L. Both A and B
ȅ M. Neither A nor B

35. In diagram “D,” which bulb(s) will light when the switch is closed?

ȅ A. A Diagram D
ȅ B. B
ȅ C. Both A and B
ȅ D. Neither A nor B

36. In diagram “E,” which bulb(s) will light when the switch is closed?

ȅ J. A Diagram E
ȅ K. B
ȅ L. Both A and B
ȅ M. Neither A nor B

4th Grade Edition 103


SCIENCE
Answer Key
Earth Science Life Science Physical Science

1. C 1. C 1. A
2. M 2. K 2. M
3. A 3. C 3. A
4. K 4. J 4. K
5. B 5. C 5. A
6. L 6. J 6. M
7. A 7. B 7. B
8. L 8. M 8. M
9. C 9. C 9. A
10. K 10. L 10. K
11. A 11. D 11. B
12. L 12. K 12. M
13. B 13. A 13. D
14. K 14. K 14. M
15. C 15. A 15. A
16. M 16. M 16. K
17. B 17. B 17. D
18. M 18. L 18. L
19. B 19. C 19. B
20. K 20. K 20. L
21. B 21. C 21. A
22. J 22. M 22. K
23. C 23. B 23. B
24. M 24. M 24. L
25. A 25. B 25. D
26. L 26. M 26. K
27. A 27. A 27. A
28. L 28. L 28. M
29. C 29. B 29. C
30. K 30. L 30. L
31. B 31. A
32. J 32. K
33. B
34. L
35. C
36. L

4th Grade Edition 104


MATH GLOSSARY
Acute Angle – An angle that has a measure of less than 90º.
Addend – A number that is added. In 8 + 4 = 12, the addends are 8 and 4.
Angle – The figure formed by two rays with the same endpoint.
Area – A number indicating the size of the inside of a plane figure.
Associative property of addition – The way in which addends are grouped does not
affect the sum. Also called the grouping property of addition.
For example: (7 + 2) + 5 = 7 + (2 + 5)
Associative property of multiplication – The way in which factors are grouped does not
affect the product. Also called the grouping property of multiplication.
For example: (7 x 2) x 5 = 7 x (2 x 5)
Average – A number obtained by dividing the sum of two or more addends by the
number of addends.
Cardinal number – A number, such as three, used to count or to tell how many.
Central angle – An angle with its vertex at the center of a circle.
Circle – A plane figure with all of its points the same distance from a given point called
the center.
Circumference – The distance around a circle.
Common denominator – A common multiple of two or more denominators. A common
denominator for 1/6 and 3/8 is 48.
Common factor – A number that is a factor of two or more numbers. A common factor
of 6 and 12 is 3.
Common multiple – A number that is a multiple of two or more numbers. A common
multiple of 4 and 6 is 12.
Commutative property of addition – The order in which numbers are added does not
affect the sum. Also called the order property of addition.
For example: 4+6=6+4
Commutative property of multiplication – The order in which numbers are multiplied
does not affect the product. Also called the order property of multiplication.
For example: 4x6=6x4
Cone – A space figure formed by connecting a circle to a point not in the plane of the
circle.
Congruent – Having the same size and the same shape.
Cube – A prism with all congruent square faces.
Decimal – A number that is written using place value and a decimal point.
Degree (of an angle) – A unit for measuring angles.
Diagonal – In a polygon, a segment that connects one vertex to another vertex but is not
a side of the polygon.
Diameter – In a circle, a segment that passes through the center and has its endpoints on
the circle.
Difference – The answer to a subtraction problem. In 95 – 68 = 27, the difference is 27.
Digit – Any of the single symbols used to write numbers. In the base-ten system, the
digits are 0,1,2,3,4,5,6,7,8, and 9.
Distributive property – The general pattern of numbers of which the following is an
example. 4 x (7 + 3) = (4 x 7) + (4 x 3)

4th Grade Edition 105


Dividend – A number that is divided by another number. In 48 ÷ 6 = 8, the dividend is
48.
Divisor – A number that divides another number. In 48 ÷ 6 = 8, the divisor is 6.
Edge – In a space figure, a segment where two faces meet.
Endpoint – The point at the end of a segment or a ray.
Equal fractions – Fractions that name the same number. 1/2 and 6/12 are equal fractions.
Equation – A mathematical sentence that uses the = symbol. 14 – 7 = 7.
Equilateral triangle – A triangle with all three sides congruent.
Even number – A whole number with a factor of 2.
Expanded form – The expanded form for 5,176 is 5,000 + 100 + 70 + 6.
Face – A flat surface that is part of a polyhedron.
Factor – (1) A number to be multiplied. (2) A number that divides evenly into a given
second number is a factor of that number.
Fraction – A number written in the form a/b, such as 2/3, or 11/5, or 4/1.
Greater than (>) – A relation between two numbers with the greater number given first.
Greatest common factor – The greatest number that is a factor of two or more numbers.
The greatest common factor of 8 and 12 is 4.
Grouping property – See Associative property of addition and Associative property of
multiplication.
Hexagon – A six-sided polygon.
Improper fraction – A fraction that names a whole number or a mixed number, such as
15/2 and 2/1.
Intersection lines – Two lines that meet at exactly one point.
Isosceles triangle – A triangle with at least two sides congruent.
Least common multiple – The smallest number that is a common multiple of two given
numbers. The least common multiple of 6 and 8 is 24.
Less than (<) – A relation between two numbers with the lesser number given first.
Line of symmetry – A fold line of a figure that makes the two parts of the figure match
exactly.
Lowest terms – A fraction is in lowest terms if 1 is the only number that will divide both
the numerator and the denominator.
Minuend – A number from which another number is subtracted. In 95 – 68 = 27, the
minuend is 95.
Mixed number – A number that has a whole number part and a fraction part, such as 3 ¼
and 6 ½.
Multiple – A multiple of a number is the product of that number and a whole number.
Some multiples of 3 are 3, 6, and 9.
Multiplicand – A number that is multiplied by another number. 7 x 3 = 21. The
Multiplicand is 7.
Multiplier – A number that multiplies another number. 7 x 3 = 21. The multiplier is 3.
Number pair – See Ordered pair.
Number sentence – An equation or an inequality. 3 + 5 = 8 4 < 7 9 > 6
Obtuse angle – An angle that has a measure greater than 90º and less than 180º.
Octagon – An eight-sided polygon.
Odd number – A whole number that does not have 2 as a factor.

4th Grade Edition 106


Order property – See Commutative property of addition and Commutative property of
multiplication.
Ordered pair – A number pair, such as (3, 5), in which 3 is the first number and 5 is the
second number.
Ordinal number – A number, such as third, used to tell order or position.
Parallel lines – Lines in the same plane that do not meet.
Parallelogram – A quadrilateral with opposite sides parallel.
Pentagon – A five-sided polygon.
Percent (%) – A word indicating “hundredths” or “out of 100.” 45 percent (45%) means
0.45 or 45/100.
Perimeter – The sum of the lengths of the sides of a polygon.
Perpendicular lines – Two intersecting lines that form right angles.
Place value – In a number, the value given to the place in which a digit appears. In 683,
6 is the hundreds place, 8 is in the tens place, and 3 is in the ones place.
Polygon – A plane figure made up of segments called its sides, each side intersecting two
other sides, one at each of its endpoints.
Prime factor – A factor that is a prime number. The prime factors of 10 are 2 and 5.
Prime number – A whole number greater than 1 that has exactly two factors: itself
and 1. 17 is a prime number.
Prism – A polyhedron with two parallel congruent faces, called bases and other faces are
parallelograms.
Probability – A number that tells how likely it is that a certain event will happen.
Product - The answer to a multiplication problem. In 3 x 7 = 21, the product is 21.
Pyramid – The space figure formed by connecting points of a polygon to a point not in
the plane of the polygon. The polygon is the base.
Quadrilateral – A four-sided polygon.
Quotient – The answer to a division problem. In 48 ÷ 6 = 8, the quotient is 8.
Radius – (1) in a circle, a segment that connects the center of the circle with a point on
the circle. (2) in a circle, the distance from the center to a point on the circle.
Ratio – A pair of numbers that expresses a rate or a comparison.
Ray – Part of a line that has one endpoint and goes on and on in one direction.
Rectangle – A parallelogram with four right angles.
Rectangular prism – See Prism.
Rectangular pyramid – See Pyramid.
Regular polygon – A polygon with all sides congruent and all angles congruent.
Right angle – An angle that has a measure 90º.
Rounded number - A number expressed to the nearest 10, 100, 1,000, and so on. 352
rounded to the nearest 10 is 350.
Scalene triangle – A triangle with no two sides congruent.
Segment – Part of line, including the two endpoints.
Similar figures – Figures with the same shape but not necessarily the same size.
Sphere – A space figure with all of its points the same distance from a given point called
the center.
Square – A rectangle with all four sides congruent.
Standard form – The standard form for 5 thousands 1 hundred 7 tens 6 ones is 5,176.

4th Grade Edition 107


Subtrahend – A number to be subtracted from another number. In 95 – 68 = 27, the
subtrahend is 68.
Sum – the answer to an addition problem. In 8 + 4 = 12, the sum is 12.
Triangle – A three-sided polygon.
Triangular prism – See Prism.
Triangular pyramid – See Pyramid.
Vertex – (1) The common endpoint of two rays that form an angle. (2) The point of
intersection of two sides of a polygon. (3) The point of intersection of the edges of a
polyhedron.
Volume – A number indicating the size of the inside of a space figure.
Whole number – One of the numbers 0, 1, 2, 3, 4, and so on.

4th Grade Edition 108


4th Grade Edition 109

S-ar putea să vă placă și